Sei sulla pagina 1di 47

P hysi cs | 3.

19

V2
vii. Radial or normal or centripetal acceleration ar = = ω2r
r

  
( )
12
viii. Total acceleration a =at + ar ,a =a2t + ar2 W
AC
tation
Ro

ix. Angular acceleration α = (non-uniform circular motion)
dt

v 2 mv 2
x. Radius of curvature =
R = Figure 3.27
a⊥ F⊥

Solved Examples

JEE Main/Boards Then,=


y x tan
= θ
2u2 tan2 θ
g
Example 1: A particle is projected horizontally with a
speed u from the top of a plane inclined at an angle θ The distance AP = l = x2 + y 2
with the horizontal. How far from the point of projection
will the particle strike the plane? 2u2 2u2
2
= tan θ 1 + tan
= θ tan θ sec θ
Sol: Take the x-axis parallel to the horizontal. Take the g g
y-axis along the vertical. Along x-axis velocity is uniform.
Along y-axis initial velocity is zero and acceleration is Examples 2: A projectile is projected at an angle 60°
uniform.
from the horizontal with a speed of
Take, X–Y axes as shown in Figure. Suppose that the time (in seconds) after which the inclination of the
(
3 + 1 m / s . The )
particle strikes the plane at point P with coordinates (x projectile with horizontal becomes 45° is:
and y). Consider the motion between A and P.
Sol: Take the x-axis along the horizontal. Take the y-axis
u vertically upwards. Along x-axis velocity is uniform.
A X
Along y-axis initial velocity is positive and acceleration
y is uniform and negative.
P Let “u” and “v” be the speed at the two specified angles.
x 
The initial components of velocities in horizontal and
Y vertical directions are:
Motion in x-direction: initial velocity = u ux = ucos60

Acceleration = 0; X = ut … (i) uy = usin60

Motion in y-direction: initial velocity = 0


Y
1
Acceleration = g; y = gt2 … (ii)
2
Eliminating t from (i) and (ii)
vy v
2
1 x 45°
y= g Also,
= y x tan θ . vx
2 u2
u
gx2 2u2 tan θ ux
Thus, = x tan θ giving x = 0, or,
2u2 g 60°
X
O uy
2
2u tan θ
Clearly the point P corresponds to x =
g
3 . 2 0 | Motion in a Plane

Similarly, the components of velocities, when the horizontal direction. Now, considering accelerated
projectile makes an angle 45 with horizontal and vertical motion in horizontal direction, we have:
directions are:
1 1g
x= R=' ux Τ + ax Τ2 ⇒ R=' ux Τ +   Τ2 ; R=' R + H

ν x =ν cos 45 ; ν y =ν sin 45 
2 22
But we know that horizontal component of velocity
remains unaltered during motion. Hence, Example 4: An airplane has to go from a point A to
another point B, 500 km away due 30° east of north. A
ucos60 wind is blowing due north at a speed of 20 m/s. The air
ν x= ux ⇒ ν cos 45= ucos60 ⇒ ν= speed of the plane is 150 m/s. (i) Find the direction in
cos 45
which the pilot should head the plane to reach the point
Here, we know initial and final velocities in vertical B. (ii) Find the time taken by the plane to go from A to B.
direction. We can apply v = u + at in vertical direction to
know the time as required: Sol: The vector sum of the velocity of the airplane with
respect to the wind and the velocity of the wind with
ν sin 45 =u + at =usin60 − gt respect to ground is equal to velocity of the aircraft with
ucos60 − ν sin 45 respect to ground. This N
⇒ ν cos
= 45 ucos60
= 
⇒t net velocity should be C R
g
in the direction A to B. 30 20m/s
Substituting value of “v” in the above equation, we have: 30 D
In the resultant 150m/s vw = 20m/s

 cos60   direction R, the plane
W
usin60 − u   X sin 45
 cos
  
45 
cos60 reaches the point B. S
⇒t= usin60 − u   X sin 45


 cos
 g 45 

⇒t= Velocity of wind V w =20 m/s
g ( 3 +1 )  ( )
3 − 1  
⇒t
=
u
g
(
sin60 − cos60= 
)
⇒t
( 310+ 1 )  ( )
32− 1 
Velocity of aero plane V a =150 m/s

⇒t
=
u
g
(
sin60 − cos60= 
)
⇒t
10  2


y
2  
⇒ t= = 0.1s
20
2
⇒ t= = 0.1s
20
Q -VA O VA P
Example 3: A projectile is at an angle “ θ ” from the
x

horizontal at the speed “u”. If an acceleration of “g/2” VBA  VB


is applied to the projectile due to wind in horizontal
direction, then find the new time of flight, maximum
height and horizontal range. R S

Sol: Take the x-axis along the horizontal. Take the y-axis
vertically upwards. Along x-axis initial velocity is positive In ∆ACD, according to the sine formula
and acceleration is uniform and positive. Along y-axis
initial velocity is positive and acceleration is uniform 20 150 20
∴ = ⇒ sinA= sin30
and negative. sinA sin30  150
20 1 1
The acceleration due to wind affects only the motion = × = ⇒ A = sin−1 (1 / 15)
in horizontal direction. It would, therefore, not affect 150 2 5
attributes like time of flight or maximum height that
results exclusively from the consideration of motion in 20
vertical direction. The generic expressions of time of R
flight, maximum height and horizontal range of flight
with acceleration are given as under: 
150
2uy u2y gΤ2 uxuy
=
Τ ; =
H = ; =
R
g 2g 4 g
(i) The direction is sin−1 (1 / 15) east of the line AB.
The expressions above revalidate the assumption made
in the beginning. We can see that it is only the horizontal (ii) sin−1 (1 / 15) = 3 48' ⇒ 30° + 3°48'= 33°48'
range that depends on the component of motion in
P hysi cs | 3.21

We note here that WR = UQ = 4 m/s


R= 1502 + 202 + 2(150)20 cos33 48'= 167m / s
Clearly, triangles ORS and ORW are congruent as two
s 500000
Time= = = 2994 sec = 49= 50min sides and one enclosed angle are equal.
v 167
WR = RS = 4 m/s; OR = OR; ∠ORW = ∠ORS = 90°
Example 5: Rain drops appear to fall in vertical direction Hence, ∠WOR = ∠SOR = 45°
to a person, who is walking at a velocity 4 m/s in a given
direction. When the person doubles his velocity in the RS RS
In triangle ORS, sin45 = ⇒ OS = = 4 2m / s
same direction, the rain drops appear to come at an OS sin45
angle of 45° from the vertical. Find the speed of the rain
drop. Example 6: A swimmer wishes to cross a 500 m wide
Sol: Velocity of rain with respect to the person is equal river flowing at 5 km/h. His speed with respect to water
to the vector sum of the velocity of rain with respect is 3 km/h. (i) If he heads in a direction making an angle
to ground and the negative of the velocity of person θ with the flow, find the time he takes to cross the river.
with respect to ground. The direction of velocity of rain (ii) Find the shortest possible time to cross the river.
with respect to person is known in each case. Assume Sol: Time taken to cross the river will depend on
a direction of velocity of rain with respect to ground the component of velocity of swimmer which is
and draw the vector diagrams for velocity of rain with perpendicular to the river flow. For shortest time this
respect to person for both the cases. component should be maximum, i.e. θ=90o.
This is a slightly tricky question. Readers may like to (i) The vertical component 3 sin θ takes him to the
visualize the problem and solve on their own before opposite side.
going through the solution given here.
Distance = 0.5 km, velocity = 3 sin θ km/h
Let us consider the situation under two cases. Here, only
the directions of relative velocities in two conditions are Dis tance 0.5km 10
=Time = = min.
given. The Figure on the left represents initial situation. Velocity 3sin θ km / h sin θ
Here, the vector OP represents velocity of the person
( VA ); OR represents relative velocity of rain drop with
respect to person ( VA ); OS represents velocity of rain 5km/h
drop. 5km/h
3km/h
The given figure represents situation when person

starts moving with double velocity. Here, the vector OT
represents velocity of the person ( VA1 ); OW represents (ii) Here vertical component of the velocity, i.e., 3 km/hr
relative velocity of rain drop with respect to person ( takes him to the opposite side (θ=90o).
VBA1 ). We should note that velocity of rain ( VB ) drop
remains the same and as such, it is represented by OS Dis tance 0.5
Time
= = = 0.16hr
represents as before. Velocity 3
∴ 0.16hr = 60 × 0.16 = 9.6 =10minute
According to the question, we are required to know the
speed of the rain drop. It means that we need to know
the angle “ θ ” and the side OS, which is the magnitude Example 7: Two tall buildings are 200 m apart. With
of velocity of rain drop. It is intuitive from the situation what speed must a ball be thrown horizontally from
that it would help if we consider the vector diagram and a window of one building 2 km above the ground so
carry out geometric analysis to find these quantities. that it will enter a window 40 m from the ground in the
For this, we substitute the vector notations with known other?
magnitudes as shown hereunder.
Sol: The time taken by ball A B
to fall from the height of
U 4 Q 4 O 4 P 4 T
X 2000 m to the height of 40 W₁ x
m (with zero initial velocity
45° 
VB
in vertical direction)
2000 m
should be equal to the
VBA1
VBA W₂
time taken by ball to cover 40 m
W 4 R 4 S a horizontal distance
of 200 m with constant 200 m
velocity in horizontal direction.

3 . 2 2 | Motion in a Plane

Figure shows the conditions of the problem. Here, A Figure shows the conditions of the problem. The boy’s
and B are the two tall buildings having windows W1 window is at O and friend’s window is at A. Let the boy
and W2, respectively. The window W1 is 2 km (=2000 m) throw the stone with a velocity v 0 making an angle θ
above the ground while window W2 is 40 m above the with the horizontal so as to enter the window at A. The
ground. We want to throw the ball from window W1 stone will follow the parabolic path with A as the highest
with such a horizontal speed ( ν x0 ) so that it enters the point on the trajectory of stone.
window W2. Note that the horizontal range of the ball R
is R = 200 m. Let t sec be the time taken by the ball ∴ =40 or horizontal range, R = 2 × 40 = 80 m
2
to reach from window W1 to window W1. This time will
depend upon the vertical motion (downward) alone. Motion in a plane

For vertical motion: V02 sin2θ V02 2sin θ cos θ


= R =
hg = 9.8ms
2000 –−2=
40 g g
= ; ν y0= 1960
0 m; g=9.8 ms-2: vy0=0
V02 sin2 θ H tan θ
1 2 2h 2 × 1960 and
= H =
∴ or
h
∴= gt or=
t = = 20 s 2g R 4
2 g 9.8
H 10 1
R 200 tan θ = 4 × = 4 × = ∴ θ = 26.56°
NowR = ν x0 t ∴ ν x0 = = = 10ms−1 R 80 2
t 20
Maximum height attained, H = 10 m
Example 8: A particle moves in a circle of radius 20 cm. Now, the projection velocity v 0 can be found by
Its linear speed is given by v = 2t; t, is in second and substituting the value of θ in formula for H.
v in metre/second. Find the radial and tangential
acceleration at t = 3 s. v 02 sin2 θ
H=
Sol: Radial acceleration depends on the square 2g
of instantaneous speed and the radius. Tangential 2 2gH 2 × 9.8 × 10
acceleration is equal to the rate of change of ∴ v=
0 = = 980 or
sin2 θ sin2 26.56°
instantaneous speed.
v0
= 980 31.3 ms−1
=
The linear speed at t = 3 s is V = 2 t = 6 m/s.
The radial acceleration at t = 3 s is
Example 10: A body is projected with a velocity of 40 ms−1 .
2 2
36m / s After 2 s, it crosses a vertical pole of height 20.4 m.
ar = ν2 / r = = 180m / s2
0.20m Calculate the angle of projection and the horizontal
The tangential acceleration is range.

dν d(2t) Sol: Use second equation of motion with constant


at
= = = 2m / s2 acceleration in vertical direction.
dt dt
Let θ be the angle of projection. For vertical motion,
Example 9: A boy wants to throw a letter wrapped
1
over a stone to his friend across the street 40 m wide. =h ( v 0 sin θ ) t − gt2
The boy’s window is 10 m below friend’s window. How 2
1
= ( 40 sin θ ) × 2 − × 9.8 × ( 2 )
2
should he throw the ball? or 20.4
2
Sol: We assume that or 20.4
= 80 sin θ − 19.6
the boy throws the ball
20.4 + 19.6 40 1
such that the maximum A ∴ sin θ= = = ∴ θ= 30°
height attained by the 80 80 2
ball is H = 10 m. This
v 0 v 02 sin2θ
Horizontal Range, R =
implies that the range H=10 m
g
of the ball is R = 40×2
( 40 ) × sin60°
2
= 80 m. Thus from the 
=
formulae of H and R we O B 9.8
can find the values of
R
=40 m = 141.1 m
initial velocity and the
2

angle of projection.
P hysi cs | 3.23

JEE Advanced/Boards  1 
(uAx t )
2
s
= +  uAy t − gt2 
 2 
Example 1: A particle A is projected with an initial 2
velocity of 60 m/s at an angle 30° to the horizontal. At
(  1
) 
2
= 30 3 × 1.09 +  30 × 1.09 − × 10 × 1.09 × 1.09 
the same time a second particle B is projected in opposite 2
 
direction with initial speed of 50 m/s from a point at a
distance of 100 m from A. If the particles collide in air, s = 62.64m
find (i) the angle of projection α of particle B, (ii) time
when the collision takes place and (iii) the distance of P Example 2: A man running on a horizontal road at
(
from A, where collision occurs. g = 10m / s2 ) 8 km/h finds the rain falling vertically. He increases his
speed to 12 km/h and finds that the drops make an
60 m/s
50 m/s angle 30° with the vertical. Find the speed and direction
of the rain with respect to the road.
30°  Sol: Velocity of rain with respect to the man is equal to
B
A
100 m the vector sum of the velocity of rain with respect to
ground and the negative of the velocity of man with
Sol: This problem is best solved in the reference frame respect to ground. The direction of velocity of rain with
of one of the two particles, say particle B. The relative respect to man is known in each case.
acceleration between the particles is zero. So in this
We have,
reference frame, the particle A moves with uniform velocity.   
v= v rain,man + v man,road  … (i)
(i) Taking x- and y-directions as shown in the figure. rain,road

Here, The two situations given in the problem may be


represented by the following diagrams.
 ˆ a =
aA =
−gj; g gjˆ Y
Vman, road O Vman, road = 12 km/h
uAx 60 cos30
= = ° 30 3m / s 8 km/h 30°
uAy 60 sin30
= = ° 30m / s  Vrain, road 
Vrain, man

V ra
30°
uBx =
−50 cos α ; uBy =
50 sin α

in,
an

ro
,m A

ad
and Relative acceleration
ain

X
Vr

between the two is zero as


 
aA = aB . Hence, the relative uAB
motion between the two is (a) (b)
uniform. It can be assumed that B is at rest and A is
 
moving with uAB . Hence, the two particles will collide, if v rain,road is same in magnitude and direction in both the

uAB is along AB. This is possible only when uAy = uBy diagram. Taking  horizontal components in E.q. (i) for the
i.e., component of relative velocity along the y-axis first diagram, v rain,road sin α =8km / h  … (ii)
should be zero. Now, consider given figure Draw a line OA ⊥ Urain,man as
30 50sin α ∴ α = sin−1 ( 3 / 5 )
Or= shown. Taking components in Eq. (i) along the line OA,
we have
(ii) Now,
V0(12)

( )
uAB = uAx − uBx = 30 3 + 50 cos α m / s
V01
 4
=  30 3 + 50 ×  m /=
 5
s (30 )
3 + 40 m / s
0
V02


AB 100
Therefore, the time of collision =
is t = 
( )

Or t = 1.09 s uAB 30 3 + 40 v rain,road sin 30o + α =12 km / h cos30o  … (iii)

From (ii) and (iii),


(iii) Distance of point P from A where collision takes
place is
3 . 2 4 | Motion in a Plane

sin ( 30° + α ) v1 ucos θ


=
12 × 3
=
sin α 8×2 At half the greatest height during upward motion,
sin30° cos α + cos30° sin α 3 3 y=
h / 2,ay =
−g,uy =
usin θ
or, =
sin α 4
Using v 2y − u2y =
2ay y
1 3 3 3 3
or, =cot α + =or,cot α h
2 2 4 2 weget, v 2y − u2 sin2 θ= 2 ( −g)
2
3 u2
sin2
θ u2 sin2 θ
or, α =cot −1 or
= v 2y u2 sin2 θ − g × =
2 2g 2
8km / h 
From(ii) v=
rain,road = 4 7km / h u2 sin2 θ  usin θ
sin α = ∴ h =  or v y
 2g  2
Example 3: Two bodies were thrown simultaneously
from the same point; one, straight up, and the other, at Hence, the resultant velocity at half of the greatest
an angle of θ= 60° to the horizontal. The initial velocity height is
of each body is equal to v 0 = 25 ms−1 . Neglecting the
air drag, find the distance between the bodies t = 1.70 s = u2 sin2 θ
v v 2x + =
v 2y u2 cos2 θ +
later. 2
v
Sol: The relative acceleration of the bodies is zero. The Given, 1 = 2
solution of this problem becomes interesting in the v2 5
frame attached with one of the bodies. 2
v u cos2 θ
2
2 1 2
= ∴ 1 = or =
Let the body thrown straight up be 1 and the other body v22
u sin θ 5 1 + 1 tan2 θ 5
2 2
be 2, then for the body 1 in the frame of 2 from the u2 cos2 θ +
2 2
kinematical equation for constant acceleration (since
2 2
both are moving under constant acceleration) is 2 + tan θ= 5,or tan θ= 3; tan θ= 3; θ= 60°
1
r12 =r0(12) + v 0(12) t + w12 t2
2
So,r12 v 0(12) t
= = ( w12 0=
and r0(12) 0) Example 5: A cannon fires successively two shells with
velocity v 0 = 250 m / s ; the first at the angle θ1 = 60°
or,=
r12 v o(12) t But, v=
01 v=
02 v0 and the second at the angle θ2 = 45° to the horizontal,
the azimuth being the same. Neglecting the air drag,
Therefore, from properties of triangle find the time interval between firings leading to the
collision of the shells.
v 0(12
= ) v 20 + v 20 − 2v 0 v 0 cos ( π / 2 − θ0 )
Sol: At the instant of y

Hence, the sought distance is collision, the horizontal


and vertical distances
r12 v 0 t 2 (1 − sin=
θ0 ) 22m covered by both the shells
v0
= v0 (x,y)
is will be equal respectively. P
Get two equations, one for ₁
₂
Example 4: The velocity of a projectile when it is at the horizontal distance and the x
greatest height is 2 / 5 times its velocity when it is at half other for vertical distance. O

of its greatest height. Determine its angle of projection. Let the shells collide at the point P(x, y). If the first shell
Sol: The maximum height is known in terms of initial takes t seconds to collide with second and ∆ t be the
velocity and angle of projection. Horizontal component time interval between the firings, then
of velocity of projectile remains constant. Use the third
equation of motion with uniform acceleration in vertical =x v 0 cos
= θ1 t v 0 cos θ2 ( t − ∆t ) ... (1)  ... (i)
direction to find the vertical component of velocity at = x v 0 cos = θ t v cos 1 θ ( t − ∆t ) 2 ... (1)
height equal to half of the maximum height. and
= y v 0 sin θ21 ( t − ∆0 t ) − 2g ( t − ∆t ) ... (2)  ... (ii)
21 
y v sin θ ( t − ∆t ) − g ( t − ∆t )
2
Suppose the particle is projected with velocity u at an and
= ... (2)
From Eq.(1)0 t = 2 ∆t cos θ2 2 ... (3)
angle θ with the horizontal. Horizontal component of its cos∆θt2cos
− cosθ2 θ1
velocity at all height will be ucos θ . From
From Eq. (i) t =
Eq.(1) ... (3) ... (iii)
From Eqs. (2) andcos(3)θ2 − cos θ1
At the greatest height, the vertical component of
velocity is zero, so the resultant velocity is= From Eqs. (2)(and
2v 0 sin θ2 )
θ1 − (3)
From
∆t Eqs. (ii) and (iii) as ∆t ≠ 0
( cos
g2v θ2(+θ1cos
0 sin − θθ21))
= ∆t as ∆t ≠ 0
= 11 sg ((on 2 + cos θ1 ) values)
cos θsubstituting
= 11 s (on substituting values)
and
= y v 0 sin θ2 ( t − ∆t ) − g ( t − ∆t ) ... (2)
2
∆t cos θ2
From Eq.(1) t = ... (3)
cos θ2 − cos θ1 P hysi cs | 3.25

From Eqs. (2) and (3)


2v 0 sin ( θ1 − θ2 ) Tangential acceleration
∆t as ∆t ≠ 0 dv
g ( cos θ2 + cos θ1 ) =α T = =k  ... (ii)
dt
= 11 s (on substituting values) v 2 k 2 t2
Normal acceleration , αN = r = r  … (iii)

Example 6: A particle A moves or αN = 4 πnk


A
along a circle of radius R = 5
(α )
1/2
cm so that its radius vector r r ∴ α= 2
+ αN2 = k 2 + 16π2n2k 2 
R T  
relative to the point O rotates 1/2
with the constant angular O 1/2
= 0.50 1 + 16 × ( 3.14 ) × ( 0.10 ) 
2 2
= k 1 + 16π2n2 
velocity ω =0.40 rads−1 . Find the    
modulus of the velocity of the = 0.8m / s2
particle, and the modulus and
direction of its acceleration.
Example 8: Two boats, A and B move away from a
Sol: Angular velocity about point O is given. We need buoy anchored at the middle of a river along mutually
to find the angular velocity about point C, ωC. Once ωC is perpendicular straight lines: the boat A along the river,
known, velocity and acceleration can be found out from and the boat B across the river. Having moved off an
formulae of circular motion. equal distance from the buoy the boats returned. Find
Angular velocity of point A, with respect to center C of τ
the ratio of times of motion of boats A if the velocity
the circle or turning rate of line CA taking the line OCX τB
d ( 2θ )  −dθ  of each boat with respect to water is n = 1.2 times
as reference line becomes ωc =− =2   =2ω
dt  dt  greater than the stream velocity.
Because angular speed of line OA is
Sol: The velocity of boat B will be vector sum of velocity
ω = −dθ / dt . of river flow and the velocity of B with respect to river.
These three vectors form a right triangle. The velocity
The turning rate of line CA is also the turning rate of of boat B is the base, the velocity of river flow is the
velocity vector of point A, which is given by v A / R . perpendicular and the velocity of B with respect to river
is the hypotenuse.
Therefore, v A =ωCR =2 ( ω) R =4 cm / s (on substituting
the values). Let l be the distance covered by the boat A along the
The acceleration of the particle will be centripetal as its river as well as by the boat B across the river. Let v 0 be
speed is constant. the stream velocity and v′ the velocity of each boat with
respect to water. Therefore, the time taken by the boat
v 2 42 A in its journey
a
= = s2 3.2 cm / s2
cm / =
R 5 l l 2lv '
tA = + =
v '+ v 0 v '− v 0 v ' 2 − v 2
0
Example 7: A point moves along a circle with a velocity
v = kt, where k = 0.5 m / s2 . Find the total acceleration And for the boat B
of the point at the moment when it has covered the
l l 2l
nth fraction of the circle after the beginning of motion, tB = + =
v '2 − v 20 v '2 − v 02 v '2 − v 02
1
where n = .
10
tA v' η
Sol: This is the case of circular motion with constant Hence,
= =
tB v ' − v 20
2 2
η −1
tangential acceleration. Use second equation of motion
with constant acceleration and zero initial velocity to  v' 
find the time required to cover 1/10 of the circle. Total  where η = 
 v0 
acceleration is the vector sum of tangential acceleration
and centripetal acceleration. tA
On substitution, = 1.8 (approx)
tB
ds 1 1
v= = kt or ∫ s0 ds =∫
k 0t t dt ∴ s = kt2
dt 2
For completion of nth fraction of the circle, Example 9: Two particles move in a uniform gravitational
field with an acceleration ‘g’. At the initial moment the
s = (2πr)n or t2= ( 4πnr ) / k  ...(i) ... (i) particles were located at one point and moved with
velocities v1 3.0
= = ms−1 and v 2 4.0 ms1 horizontally in
3 . 2 6 | Motion in a Plane

opposite directions. Find the distance between the Both the particles are initially at the same position and
particles at the moment when their velocity vectors have same acceleration g, so
become mutually perpendicular.
r0 0, =
= w 0, and v=
0 v1 − v 2 .
Sol: The relative acceleration between the particles is
Thus, the sought distance is
zero. Initial relative distance is zero. So the final relative
distance between them is equal to product of time r
= v 0=t ( v1 + v 2 ) t (using the value of t)
and relative velocity. The time required can be found
by using the equations of final velocities in Cartesian v1 + v 2
= v1 v 2
coordinates. g
= 2.5 m, on substituting the values of v1 , v 2 and g.
v₂ v₁
Example 10: A boy whirls a stone in a horizontal circle
of radius 1.5 m and at height 2.0 m above level ground.
The string breaks, and the stone flies off horizontally and
v’₂ v’₁ strikes the ground after travelling a horizontal distance
of 10 m. What is the magnitude of the centripetal
v₂t v₁t
acceleration of the stone while in circular motion?
Let the velocities of the particles (say v '1 and v '2 ) Sol: The time of fall of the stone depends on the
become mutually perpendicular after time t. Then, their height of the stone and can be found using the second
velocities become equation of motion with constant acceleration and zero
initial velocity. The horizontal component of stone’s
v1ˆi + gt ˆj ; v 2, =
v1, = − v 2ˆi + gt ˆj
velocity remains constant is equal to the horizontal
As v1, ⊥ v 2, , so, v1, ⋅ v 2, =
0 distance covered by the stone divided by the time of
fall. The centripetal acceleration is equal to the square
( )( )
or v1ˆi + gt ˆj ⋅ v 2ˆi + gt ˆj =
0 of the horizontal velocity divided by the radius of the
2 2 horizontal circle.
or − v1 v 2 + g t =
0
v1 v 2 2h 2×2 10
=t = v = 15.63m / s
= 0.64 s ; =
Hence, t = g 9.8 t
g
In the frame attached with 2 for the particle 1 v2
a
= = 163m / s2
R
1
r =r0 + v 0 t + wt2
2

JEE Main/Boards

Exercise 1 Q.3 What is a projectile? Give its examples. Show that


the path of projectile is a parabolic path when projected
horizontally from a certain height.
Projectile Motion
Q.4 Show that there are two angles of projection for
Q.1 What do you understand by motion in two which the horizontal range is the same.
dimensions? When an object is moving with uniform
velocity in two dimensions, explain displacement,
Q.5 Find (i) time of flight, (ii) Max. height and (iii)
velocity and find the equations of motion of the object.
horizontal range of projectile projected with speed
v AB ( velocity of A with respect to B=
) v A − vB
Q.2 Find the relation for (i) velocity and time (ii)
displacement and time, when an object is moving with aAB ( acceleration of A with respect to B=
) aA − aB
uniform acceleration in two dimensions.
making an angle θ with the horizontal direction from
ground.
P hysi cs | 3.27

Q.6 Find the magnitude and direction of the velocity of


where î is a unit vector along horizontal and ˆj is unit
an object at any instant during the oblique projection
vector vertically upward. Find the Cartesian equation of
of a projectile. 2
its path. (g = 10 m s )
Q.7 Find (i) the path of projectile, (ii) time of flight, (iii)
horizontal range and (iv) maximum height, when a Q.18 Find the maximum horizontal range of a cricket
projectile is projected with velocity υ making an angle θ ball projected with a velocity of 80 m/s. If the ball is
with the vertical direction. to have a range of 100 3 m, find the least angle of
projection and the least time taken.
Q.8 What is centripetal acceleration? Find its magnitude
and direction in case of a UCM of an object. Q.19 A bullet fired from a rifle attains a maximum height
of 5m and crosses a range of 200 m. Find the angle of
Q.9 A stone is dropped from the window of a bus projection.
moving at 60 kmh−1 . If the window is 196 cm. high,
find the distance along the track which the stone moves Q.20 A target is fixed on the top of a pole 13 m high.
before striking the ground. A person standing at a distance 50 m from the pole is
capable of projecting a stone with a velocity 10 g m/s.
Q.10 A hiker stands on the edge of a cliff 490 m above If he wants to strike the target in shortest possible time,
the ground and throws a stone horizontally with an at what angle should he project the stone.
initial speed of 15 ms−1 . Neglecting air resistance, find
the time taken by the stone to reach the ground, and Q.21 A particle is projected with a velocity u so that its
the speed with which it hits the ground. Take g = 9.8 horizontal range is twice the greatest height attained.
Find the horizontal range of it.
m / s2 .

Q.22 A drunkard walking in a narrow lane takes 5 steps


Q.11 A ball is thrown horizontally from the top of a
forward and 3 steps backward, and so on. Each step
tower with a speed of 50 ms−1 . Find the velocity and is 1 m long and required 1s. Determine how long the
position at the end of 3 second g = 9.8 ms−2 . drunkard takes to fall in a pit 13 m away from the start.

Q.12 A body is projected downward at an angle of 30° Q.23 A jet airplane travelling at the speed of 500km
to the horizontal with a velocity of 9.8 m/s from the top h−1 ejects its projects of combustion at the speed of
of a tower 29.4 m high. How long will it take before
1500km h−1 relative to the jet plane. What is the speed
striking the ground?
of the later with respect to observer on the ground.

Q.13 Prove that a gun will shoot three times as high


Q.24 A car moving along a straight highway with speed
when its angle of elevation is 60° as when it is 30° , but
cover the same horizontal range. of 126 km h−1 is brought to a stop within a distance of
200 m. What is the retardation of the car (assumed
uniform) and how long does it take for the car to stop?
Q.14 Prove that the maximum horizontal range is 4
times the maximum height attained by a projectile
which is fired along the required oblique direction. Q.25 Two trains A and B of length 400 m each are moving
on two parallel tracks with a uniform speed of 72km h−1
Q.15 Two particles are projected from the ground in the same direction with A head of B. The driver of B
simultaneously with speeds of 30 m/s and 20 m/s at decides to overtake A and accelerate by 1 ms−2 . If after
angles 60° and 30° with the horizontal on the same 50 s, the guard of B just brushes past the driver of A,
direction. Find maximum distance between them on what was the original distance between them?
2
ground where they strike. g = 10 m s .
Q.26 Two towns A and B are connected by a regular bus
Q.16 A projectile has the same range when the maximum service with a bus leaving in either direction every T min.
A man cycling with a speed of 20km/h in the direction
height attained by it is either H1 or H2 . Find the relation
of A and B notices that a bus goes past him every 18
between R, H1 and H2 . min in the direction of his motion, and every 6 min in
the opposite direction. What is the period T of the bus
(
Q.17 A projectile is given an initial velocity of ˆi + 2ˆj , ) service and with what speed (assumed constant) do the
buses ply on the road?
3 . 2 8 | Motion in a Plane

Q.27 A ball is dropped from a height of 90 m on a floor. he applies brakes and reduces his speed at a constant
At each collision with the floor, the ball loses one tenth −1
rate of 0.5 ms per second. Find the magnitude of the
of its speed. Plot the speed-time graph of its motion net acceleration of the cyclist.
between t=0 to 12s. (g=10 ms−2 )
Q.38 A particle moves in a circle Y
Q.28 A man walks on a straight road from his home to of radius 4.0 cm clockwise at Q
a market 2.5 km away with a speed of 5 km/h. Finding constant speed of 2 cm s−1 . If
the market closed, he instantly turns and walks back
x̂ and ŷ are unit acceleration
with a speed of 7.5 km/h. What is the (a) magnitude of X
vectors along X-axis and O P
average velocity and (b) average speed of the man, over
Y-axis respectively, find the
the interval of time (i) 0 to 30 min. (ii) 0 to 50 min (iii) 0
acceleration of the particle at
to 40 min?
the instant half way between PQ figure.

Q.29 A dive bomber, diving at an angle of 53° with the


Q.39 Three girls skating on a Q
vertical, released a bomb at an altitude of 2400ft. the
circular ice ground of radius
bomb hits the ground 5.0 s after being released. (i) What
200m start from a point P on the
is the speed of the bomber? (ii) How far did the bomb
edge of the ground and reach a
travel horizontally during its flight? (iii) What were the A B
point Q diametrically opposite C
horizontal and vertical components of its velocity just
to P following different paths
before striking the ground?
as shown in figure. What is the
magnitude of the displacement
Circular Motion vector for each? For which girl is P
this equal to the actual length of
Q.30 Calculate the angular velocity of the minute’s hand path skated?
of a clock.
Q.40 A cyclist starts from the centre O of a circular park
Q.31 What is the angular velocity in radian per second of radius 1m reaches the edge P of the park, then cycles
of a fly wheel making 300 r.p.m.? along the circumference, and returns to the centre
along QO. If the round trip takes 10 minutes, what is the
Q.32 The wheel of an automobile is rotating with 4 (i) net displacement (ii) average velocity and (iii) average
rotations per second. Find its angular velocity. If the speed of the cyclist?
radius of the fly wheel is 50cm, find the linear velocity of
a point on its circumference. Q.41 A cyclist is riding with a speed of 36 km h−1 . As he
approaches a circular turn on the road of radius 140m,
Q.33 The angular velocity of a particle moving in a he applies break and reduces his speed at the constant
circle of radius 50 cm is increased in 5 minutes from 100
rate of 1m s−2 . What is the magnitude and direction of
revolutions per minute. Find (a) angular acceleration (b)
the net acceleration of the cyclist on the circular turn?
linear acceleration.
Q.42 A particle is moving in a
Q.34 A body is moving in a circle of radius 100 cm with
circle of radius r centres at O
a time period of 2 second. Find the acceleration.
with constant speed υ . What is O 
the change in velocity in moving
Q.35 An insect trapped in a circular groove of radius from A to B? In the figure. Given 40°
12cm moves along the grove steadily and completes 7
B
∠AOB = 40° . 
revolutions in 100s. (i) What is the angular speed, and A
the linear speed of the motion? (ii) Is the acceleration
vector a constant vector? What is its magnitude?
A
Q.43 A particle originally at rest
Q.36 Calculate the centripetal acceleration of a point on h
B
at the highest point of a smooth R
the equator of earth due to the rotation of earth due  
vertical circle of radius R, is slightly
to the rotation of earth about its own axis. Radius of 
displaced. Find the vertical distance
O mg
earth=6400 km.
below the highest point where the
particle will leave the circle.
Q.37 A cyclist is riding with a speed of 27 kmh−1 . As he
approaches a circular turn on the road of radius 80.0m,
P hysi cs | 3.29

Exercise 2 Q.7 A projectile is fired with a velocity at right angles


to the slope which is inclined at an angle θ with the
horizontal. The expression for the range R along the
Projectile Motion
incline is
Single Correct Choice Type 2v 2 2v 2
(A) sec θ (B) tan θ
g g
Q.1 A particle is projected with a certain velocity at an
angle θ above the horizontal from the foot of a given 2v 2 v2
(C) tan θ sec θ (D) tan2 θ
plane inclined at an angle of 45° to the horizontal. If the g g
particle strikes the plane normally, then equals
(A) tan−1 (1 3) (B) tan−1 (1 2) Q.8 A hunter tries to hunt a monkey with a small, very
poisonous arrow, blown from a pipe with initial speed
(C) tan−1 (1 2) (D) tan−1 3 v 0 . The monkey is hanging on a branch of a tree at
height H above the ground. The hunter is at a distance L
Q.2. Two projectiles A and B are thrown with the same from the bottom of the tree. The monkey sees the arrow
such that A makes angle θ with the horizontal and B leaving the blow pipe and immediately lose the grip
makes angle θ with the vertical, then on the tree, falling freely down with zero initial velocity.
(A) Both must have same time of fight The minimum initial speed v 0 of the arrow for hunter to
succeed while monkey is in air
(B) Both must achieve same maximum height
(C) A must have more horizontal range than B
(A)
(
g H2 + L2 ) (B)
gH2

(D) Both may have same time of flight 2H H2 + L2

Q.3 A projectile is fired with a speed u at an angle θ with


(C)
(
g H2 + L2 ) (D)
2gH2
the horizontal. Its speed when its direction of motion H H2 + L2
makes an angle ' α ' with the horizontal is
Q.9 A swimmer swims in still water at a speed=5 km/hr.
(A) usec θ cos α (B) usec θ sin α He enters a 200 m wide river, having river flow speed=4
(C) ucos θ sec α (D) usin θ sec α km/hr at point A and proceeds to swim at an angle of
127° with the river flow direction. Another point B is
Q.4 A ball is projectile from top of tower with a velocity located directly across A on the other side. The swimmer
of 5 m/s at an angle of 53° to horizontal. Its speed when lands on the other bank at a point C, from which he
it is at a height of 0.45m from the point of projection is: walks the distance CB with a speed=3 km/hr. The total
time in which he reaches from A to B is
(A) 2 m/s (B) 3 m/s
(A) 5 minutes (B) 4 minutes
(C) 4 m/s (D) Data insufficient
(C) 3 minutes (D) None

Q.5 particle is dropped from the height of 20m from Q.10 A boat having a speed of 5 km/hr. in still water,
horizontal ground. A constant force acts on the crossed a river of width 1 km along the shortest possible
particle in horizontal direction due to which horizontal path in 15 minutes. The speed of the river in Km/hr.
acceleration of the particle becomes 6 ms-2. Find the
horizontal displacement of the particle till it reaches (A) 1 (B) 3 (C) 4 (D) 41
ground.
Q.11 A motor boat is to reach at a point 30° upstream
(A) 6m (B) 10 m (C) 12 m (D) 24 m
(w.r.t. normal) on other side of a river flowing with
velocity 5m/s. Velocity of motorboat w.r.t. water is 5 3
Q.6 Find time of flight of projectile thrown horizontally m/s. The driver should steer the boat at an angle
−1
with speed 10 ms from a long inclined plane which (A) 120° w.r.t. stream direction
makes an angle of θ= 45° from horizontal.
(B) 30° w.r.t. normal to the bank
(A) 2 sec (B) 2 2 sec (C) 30° w.r.t. the line of destination from starting point.
(C) 2 sec (D) None of these (D) None of these
3 . 3 0 | Motion in a Plane

Q.12 A flag is mounted on a car moving due north with Q.18 If acceleration of A is 2 m/s2 to left and acceleration
velocity of 20 km/hr. Strong winds are blowing due East of B is 1 m/s2 to left, then acceleration C is
with velocity of 20km/hr. The flag will point in direction A B
(A) East (B) North-East
(C) South-East (D) South-West

Q.13 Three ships A, B & C are in motion. The motion of C


A as seen by B is with speed v towards north-east. The
motion of B as seen by C is with speed v towards the (A) 1 m/s2 upwards (B) 1 m/s2 downwards
north-west. Then as seen by A, C will be moving towards
(C) 2 m/s2 downwards (D) 2m/s2 upward
(A) north (B) south (C) east (D) west

Circular Motion
Q.14 Wind is blowing in the north direction at speed of
2 m/s which causes the rain to fall at some angle with Single Correct Choice Type
the vertical. With what velocity should a cyclist drive so
that the rain appears vertical to him? Q.19 Two racing cars of masses m1 and m2 are moving
(A) 2m/s south (B) 2m/s north in circles of radii r1 and r2 respectively, their speeds are
such that they each make a complete circle in the same
(C) 4m/s west (D) 4 m/s south
time t. The ratio of the angular speed of the first to the
second car is:
Q.15 When the driver of a car A sees a car B moving
towards his car and at a distance 30m, takes a left turn (A) m1 : m2 (B) r1 : r2 (C) 1:1 (D) m1r1 : m2r2
of 30° . At the same instant the driver of the car B takes
a turn to his right at an angle 60° . The two cars collides Q.20 A particle moves in a circle of radius 25 cm at two
after two seconds, then the velocity (in m/s) of the car revolution per sec. The acceleration of the particle in
A and B respectively will be : [assume both cars to be m / s2 is:
moving along same line with constant speed] (A) π2 (B) 8π2 (C) 4π2 (D) 2π2
(A) 7.5, 7.5 3 (B) 7.5, 7.5
Q.21 Two particle and Q are located at distance rP and
(C) 7.5 3 , 7.5 (D) None
rQ respectively from the centre of a rotating disc such
that rP > rQ :
Q.16 At a given instant, A is moving
with velocity of 5m/s upwards. What is (A) Both P and Q have the same acceleration
velocity of B at that time (B) Both P and Q do not have any acceleration
(A) 15 m/s ↓ (C) P has greater acceleration than Q
A
(B) 15 m/s ↑ B (D) Q has greater acceleration than P
(C) 5 m/s ↓
(D) 5 m/s ↑ Q.22 When particle moves in a circle with a uniform
speed:
Q.17 The pulleys in the diagram (A) Its velocity and acceleration are both constant
are all smooth and light. The (B) Its velocity is constant but the acceleration changes
acceleration of A us a upwards
and the acceleration of A is a (C) Its acceleration is constant but the velocity changes
upward and the acceleration A (D) Its velocity and acceleration both change
of C is f downwards. The C
acceleration is Q.23 If a particle moves in a circle described equal
(A) 1 2(f − a) up B angles in equal times, its velocity vector:

(B) 1 2(a+ f) down (A) Remains constant


(B) Changes in magnitude
(C) 1 2(a+ f) up
(C) Changes in direction
(D) 1 2(a− f) up
(D) Changes both in magnitude and direction
P hysi cs | 3.31

Q.24 If the equation for the displacement of a particle Q.2 A boat which has a speed of 5 km/h in still water
moving on a circular path is given by ( θ= crosses a river of width 1 km along the shortest possible
) 2t + 0.5 ,
3
path in 15 mi. The velocity of the river water in km/h is
where θ is in radians and t in seconds, then the angular
velocity of the particle after 2 sec from its start is: (1988)

(A) 8 rad/sec (B) 12 rad/sec Assertion Reasoning Type


(C) 24 rad/sec (D) 36 rad/sec
(A) If statement-I is true, statement-II is true:
Q.25 The second’s hand of a watch has length 6cm. statement-II is the correct explanation for statement-I
Speed of end point and magnitude of difference of (B) if statement-I is true, Statement-II is true: statement‑II
velocities at two perpendicular ll be: is not a correct explanation of statement-I
(A) 6.28 & 0 mm/s (B) 8.88 & 4.44 mm/s (C) If statement-I is true: statement-II is false
(C) 8.88 & 6.28 m/s (D) 6.28 & 8.88 mm/s (D) If Statement-I is false: statement-II is true

Q.26 A fan is making 600 revolutions per minute. If after


some time it makes 1200 revolutions per minute, then Q.3 Statement-I: For an observer looking out through
the window of a fast moving train, the nearby objects
increases in its angular velocity is:
appear to move in the opposite direction to the train,
(A) 10π rad/sec (B) 20π rad/sec while the distant objects appear to be stationary.
(C) 40π rad/sec (D) 60π rad/sec Statement-II: If the observer and the object are moving

at velocities v 1 and v 2 respectively with reference to a
Q.27 A wheel completes 2000 revolutions to cover the
laboratory frame, the
 velocity of the object with respect
9.5 km. distance, then the diameter of the wheel is: 
to the observer is v 2 − v 1 (2008)
(A) 1.5 m (B) 1.5 cm (C) 7.5 cm (D) 7.5 m
Q.4 For a particle in uniform circular motion the
Q.28 A body moves with constant angular velocity on a 
circle. Magnitude of angular acceleration is: acceleration a at a point P (R, θ ) on the circle of radius
R is (here θ is measures from the x-axis)  (2010)
2
(A) rω (B) Constant
v2 v2 v2 v2
(C) Zero (D) None of the above (A) − cos θˆi + sin θˆj (B) − sin θˆi + sin θˆj
R R R R
Q.29 For a particle in a uniformly accelerated (speed v2 v2 v2 ˆ v2 ˆ
increasing uniformly) circular motion: (C) − cos θˆi − sin θˆj (D) i+ j
R R R R
(A) Velocity is radial and acceleration is transverse only.
(B) Velocity is transverse and acceleration is radial only Q.5 A boy can throw a stone up to a maximum height
(C) Velocity is radial and acceleration has both radial of 10 m. The maximum horizontal distance that the boy
and transverse components can throw the same stone up to will be (2012)
(D) Velocity is transverse and acceleration has both
(A) 20 2 m (B) 10 m (C) 10 2 m (D) 20 m
radial and transverse components

Q.30 A particle moves in a circular orbit under the Q.6 Two cars of masses m1 and m2 are moving in circles
force proportional to the distance ‘r’. The speed of the of radii r1 and r2, respectively. Their speeds are such that
particle is: they make complete circles in the same time t. The ratio
of their centripetal acceleration is (2012)
(A) Proportional of r 2 (B) Independent of r
(A) m1r1: m2r2 (B) m1 : m2 (C) r1 : r2 (D) 1 : 1
(C) Proportional to r (D) Proportional to 1/r

Q.7 A projectile is given an initial velocity of ˆi + 2ˆj ( )


Previous Years’ Questions m/s, where î is along the ground and ˆj is along the
vertical. If g = 10 m/s2, the equation of its trajectory is:
Q.1 A river is flowing from west to east at a speed of 5 (2013)
m/min. A man on the south bank of the river, capable
of swimming at 10m/min in still water, wants to swim (A) y = 2x − 5x2 (B) 4y = 2x − 5x2
across the river in the shortest time. He should swim in (C) 4y = 2x − 25x2 (D) y = x – 5x2
a direction (1983)
3 . 3 2 | Motion in a Plane

JEE Advanced/Boards

Exercise 1 Q.7 A shell is fired from a point O at an angle of 60°


with a speed of 40 m/s & it strikes a horizontal plane
through O. at a point A. The gun is fired a second time
Projectile Motion
with the same angle of elevation but a different speed
v. If it hits the target which starts to rise 9 3 m/s at the
Q.1 A particle moves in the plane XY with constant
same instant as the shell is fired, find v. (Take g=10 m/
acceleration a directed along the negative y axis. The
equation of motion of the particle has the form y = αx − βx2 , s2 )
where α and β are positive constants. Find the velocity
of the particle at the origin of coordinates. Q.8 A cricket ball thrown from a height of 1.8 m at an
angle of 30° with the horizontal at a speed of 18 m/s is
Q.2 Two seconds after projection, a projectile is moving caught by another field’s man at a height of 0.6 m from
at 30° above the horizontal, after one more second it is the ground. How far were the two men apart?
moving horizontally. Find the magnitude and direction
of its initial velocity. (g=10 m/ s2 ) Q.9 A batsman hits the ball at a height 4.0 ft. from the
ground at projection angle of 45° and the horizontal
Q.3 A ball is projected from O with an initial velocity 700 range is 350 ft. Ball falls on left boundary line, where a
cm/sec in a direction 37° above the horizontal. A ball B, 24 ft height fence is situated at a distance of 320 ft. Will
500 cm away from O on the line of the initial velocity of the ball clear the fence? u
P

A, is released from rest at the instant A is projected. Find


Q.10 (i) A particle is projected
(i) The height through which B falls, before it is hit by A. with a velocity of 29.4 m/s at an h
(ii) The direction of the velocity A at the time of impact angle of 60° to the horizontal.
(Given g=10 m s−2 , sin 37° =0.6) Find the range on a plane

inclined at 30° to the horizontal
when projected from a point of
Q.4 On a frictionless horizontal surface, assumed to be
the plane up the plane.
the x-y plane, a small trolley A is moving along a straight
line parallel to the y-axis with a constant velocity of (ii) determine the velocity with which a stone must be
( )
3 − 1 m/s. At a particular instant, when the line OA projected horizontally from a point P, so that it may hit
the inclined plane perpendicularly. The inclination of the
makes an angle 45° with the x-axis, a ball is thrown
plane with the horizontal is θ and P is h metre above the
along the surface from the origin O. Its velocity makes
foot of the incline as shown in the Figure.
an angle φ with the x-axis and it hits the trolley. y
(i) The motion of the ball is observed from the frame of Q.11 During the
the trolley. Calculate the angle θ made by the velocity volcanic eruption
vector of the ball with the x-axis in this frame. chunks of solid rock
(ii) Find the speed of the ball with respect to the surface are blasted out of the A 37°
x
if φ = 4 θ 3. volcano.
3.3km

(i) At what initial speed B


Q.5 If R us the horizontal range and u would a volcanic object 9.4km
h, the greatest height of a projectile, h have to be ejected at
find the initial speed. g = 10m s 
2 37° to the horizontal
R from the vent A in
order to fall at B as shown in Figure.
(ii) What is the time of flight. (g=9.8 m/ s2 )
Q.6 A stone is thrown horizontally from a tower. In 0.5
second after the stone began to move, the numerical Q.12 A projectile is projected with an initial velocity of
value of its velocity was 1.5 times its initial velocity. Find
the initial velocity of stone. ( )
6iˆ + 8ˆj ms−1, ˆi =unit vector in horizontal direction and ˆj =
unit vector in vertical upward direction then calculate
its horizontal range, maximum height and time of flight.
P hysi cs | 3.33

Q.13 An aeroplane is flying at a height of 1960 metre in Q.21 A butterfly is flying with velocity y
a horizontal direction with a velocity of 100 m/s, when 10iˆ + 12ˆj m/s and wind is blowing B
it is vertically above an object M on the ground it drops along x axis with velocity u. If butterfly
a bomb. If the bomb reaches the ground at the point N, starts motion from A and after some A 37°
then calculate the time taken by the bomb to reach the time reaches point B, find the value of u. x
ground and also find the distance MN.

Q.14 A projectile is projected from the base of a hill Q.22 Rain is falling vertically with a speed of 20 m/ s−1
whose slope is that of right circular cone, whose axis is relative to air. A person is running in the rain with a
vertical. The projectile grazes the vertex and strikes the velocity of 5 m/ s−1 and a wind is also blowing with a
hill again at a point on the base. If θ be the semi-vertical
speed of 15 m/ s−1 (both towards east). Find the angle
angle of the cone, h its height u the initial velocity of
with the vertical at which the person should hold his
projection and α the angle of projection, show that
umbrella so that he may not get drenched.
gh(4 + tan2 θ)
(i) tan
= θ 2cot α (ii) u2 =
2 Circular Motion
Q.15 A person is standing on a truck moving with a
constant velocity of 14.7 m/s on a horizontal road. The Q.23 A bullet is moving horizontally with certain
man throws a ball in such a way that it returns to the velocity. It pierces two paper discs rotating co-axially
truck after the truck has moved 58.8 m. Find the speed with angular speed ω separated by a distance  . If the
and the angle of projection (i) as seen from the truck, (ii) hole made by the bullet on 2nd disc is shifted through
as seen from the road. an angle θ with respect to that in the first disc, find the
velocity of the bullet, (change of velocity in the bullet is
neglected)
Q.16 Two bodies are thrown simultaneously from the
same point. One thrown straight up and the other at an
Q.24 Position vector of a particle performing circular
angle α with the horizontal. Both the bodies have equal
velocity of v 0 Neglecting air drag, find the separation of motion is given by = r 3iˆ + 4ˆj m, velocity vector is
 
the particle at time t. v= −4iˆ + 3ˆj m/s. If acceleration is a =−7iˆ − ˆj m/ s2 find
the radial and tangential components of acceleration.
Q.17 Two particles move in a uniform gravitational field
with an acceleration g. At the initial moment the particles Q.25 An astronaut is rotating in a rotor having vertical
were located at one point and moved with velocities axis and radius 4m. If he can withstand upto acceleration
3 m/s and 4 m/s horizontally in opposite directions. Find of 10g. Then what is the maximum number of permissible
the distance between the particles at the moment when (
revolution per second? g = m s2 . )
their velocity vectors become mutually perpendicular.
Q.26 A racing-car of 1000 kg moves round a banked
Q.18 A particle is projected from O at an elevation α track at a constant speed of 108 km h−1 . Assuming the
and after t second it has an elevation β as seen from total reaction at the wheels is normal to the track and
the point of projection. Prove that its initial velocity is the horizontal radius of the track is 90 m, calculate the
gt cos β angle of inclination of the track to the horizontal and
. the reaction at the wheels.
sin(α − β)
Q.27 A particle A moves along a A
Q.19 The velocity of a particle when it is at its greatest circle of radius R=50 cm so that its r
2 radius vector r relative to the point O R
height is of its velocity when it is at half its greatest (see figure) rotates with the constant O
5 C
angular velocity ω =0.40 rad/sec.
height. Find the angle of projection of the particle. Find the modulus of the velocity
of the particle and modulus and
Q.20 A man crosses a river in a boat. If he crosses the direction of its total acceleration.
river in minimum time he takes 10 minutes with a drift
120 m. If he crosses the river taking shortest path, he Q.28 A wet open umbrella is held upright and is rotated
takes 12.5 minutes. Assuming v b r > v r find about the handle at uniform rate of 21 revolutions is 44s.
(i) Width of the river If the rim of the umbrella circle is 1 meter in diameter
(ii) Velocity of the boat with respect to water, and the height of the rim above the floor is 1.5m, find
where the drops of water spun off the rim and hit the
(iii) Speed of the current.
floor.
3 . 3 4 | Motion in a Plane

Q.29 A spaceman in training is rotated in a seat at the (A) The maximum height reached by the projectile can
end of horizontal rotating arm of length 5m. if he can be 0.25 m.
withstand acceleration up to 9 g, what is the maximum (B) The minimum velocity during its motion can be
number of revolutions per second permissible? Take
15 m/s
g = 10m s2
3
(C) The time taken for the flight can be sec.
Q.30 An insect on the axle of a wheel observes the 5
motion of a particle and ‘find’ it to take its place along (D) Minimum kinetic energy during its motion can be 6J.
the circumference of a circle of radius ‘R’ with a uniform
angular speed ω . The axle is moving with a uniform
speed ‘v’ relative to the ground. How will an observer Q.2 A particle is projected from the ground with velocity
on the ground describe the motion of the same point? u at angle θ with horizontal. The horizontal range,
maximum height and time of fight are R , H and T
Q.31 A stone is thrown horizontally with a velocity 10 u2 sin2θ u2 sin2 θ
m/s. Find the radius of curvature of its trajectory in 3 respectively. They are given
= by, R = ,H
g 2g
second after the motion began. Disregard the resistance 2usin θ
of air. and T =
g

Q.32 A table with smooth horizontal surface is fixed Now keeping u as fixed, θ is varied from 30° to 60° .
Then,
in a cabin that rotates with a uniform angular velocity
ω =220s−1 in a circular path of radius R=700m. A smooth (A) R will first increase then decrease, H will increase and
groove AB of length L=7 m is made on the surface of T will decrease
the table. The groove makes an angle θ= 30° with the (B) R will first increase then decrease while H and T both
radius OA of the circle in which the cabin rotates. A will increase
small particle is kept at the point A in the groove and is
(C) R will first decrease while H and T will increase
released to move along AB. Find the time taken by the
particle to reach the point B. (D) R will first increase while H and T will increase

B Q.3 The trajectory of particle 1 with respect to particle


O A  2 will be

R (A) A parabola (B) A straight line



(C) A vertical straight line (D) A horizontal straight line

Q.33 A smooth sphere of radius R is made to translate in Q.4 If v1 cos=


θ1 v 2 cos θ2 , then choose the incorrect
a straight line with a constant acceleration a. A particle statement.
kept on the top of the sphere is released from there at (A) One particle will remain exactly below of above the
zero velocity with respect to the sphere. Find the speed other particle
of the particle with respect to the sphere as a function
(B) The trajectory of one with respect to other will be a
of the angle θ slides.
vertical straight line
Q.34 If a particle is rotating in a circle of radius R with (C) Both will have the same range
velocity at an instant v and the tangential acceleration is
(D) None of these
a. Find the net acceleration of the particle.

Q.5 If v1 sin=
θ1 v 2 sin θ2 , then choose the incorrect
Exercise 2 statement.
(A) The time of flight of both the particle will be same
Projectile Motion
(B) The maximum height attained by the particles will
Single Correct Choice Type be same
(C) The trajectory of one with respect to another will be
Q.1 A projectile of mass 1 kg is projected with a velocity a horizontal straight line
of 20 m/s such that it strikes on the same level as the (D) None of these
point of projection at a distance of 3 m. Which of the
following options is incorrect?
P hysi cs | 3.35

Multiple Correct Choice Type (B) P will hit the roof BC, if a>g B C
(C) P will hit the wall CD if a<g
Q.6 Choose the correct alternative (s) u
(D) May be either (A), (B) or (C), P
(A) If the greatest height to which a man can throw a depending on the speed of projection A D
stone is h, then the greatest horizontal distance upto of P
which he can throw the stone is 2h.
(B) The angle of projection for a projectile motion whose Q.11 The vertical height of point P above the ground is
range R is m times the maximum height is tan−1 (4/n) twice that of point Q. A particle is projected downward
with a speed of 5 m/s from P and at the same time another
(C) The time of flight T and the horizontal range R of a particle is projected upward with the same speed from
projectile are connected by the equation= gT2 2R tan θ Q. Both particles reach the ground simultaneously, if PQ
where θ is the angle of projection. is lie on same vertical line then
(D) A ball is thrown vertically up. Another ball is thrown (A) PQ=30 m
at an angle θ with the vertical. Both of them remain in
air for the same period of time. Then the ratio of heights (B) PQ=60 m
attained by the two balls 1:1. (C) Time of flight of the stones
(D) Time of flight of the stones=1/3s
Q.7 If it is the total time of flight, h is the maximum
height & R is the range for horizontal motion, the x & y
co-ordinates of projectile motion and time t are related Q.12 Two particles A & B projected along different
as: directions from the same point P on the ground with the
same speed of 70 m/s in the same vertical plane. They
 t  t  X  X hit the ground at the same point Q such that PQ=480m.
(A) y 4th  T   1 − T  =
= (B) y 4th    1 − 
   R
  R  Then: (Use g=9.8 m/ s2 , sin−1 0.96= 74° , sin−1 0.6
= 37° )
 T  T  R  R (A) Ratio of their times of flights is 4:5
(D) y 4th    1 − 
(C) y 4th    1 −  =
=
 t  t X
  X
(B) Ratio of their maximum heights is 9:16
(C) Ratio of their minimum speeds during flight is 4:3
Q.8 A particle moves in the xy plane with a constant
acceleration ‘g’ in the negative y-direction. Its equation (D) The bisector of the angle between their directions
of motion is =y ax − bx2 , where a and b are constants. of projection makes 45° with horizontal.
Which of the following is correct?
Comprehension Type
(A) The x-components of its velocity is constant.
(B) At the origin, the y-component of its velocity is a A projectile is thrown with a velocity of 50m s−1 at an
angle of 53° with the horizontal.
g .
2b
(C) At the origin, its velocity makes an angle tan−1 (a) Q.13 Choose the incorrect statement
with the x-axis. (A) It travels vertically with a velocity of 40m s−1
(D) The particle moves exactly like a projectile.
(B) It travels horizontally with a velocity of 30m s−1
Q.9 A ball is rolled off along the edge of a horizontal (C) The minimum velocity of the projectile is 30 m s−1
table with velocity 4m/s. It hits the ground after time
0.4s. Which of the following are correct? (D) None of these

(A) The height of the table is 0.8 m. Q.14 Determine the instants at which the projectile is at
(B) It hits the ground at an angle of 60° with the vertical the same height.
(C) It covers a horizontal distance 1.6 m from the table (A) t=1s and t=7s (B) t=3s and t=5s
(D) It hits the ground with vertical velocity 4m/s (C) t=2s and t=6s (D) all the above

Q.10 A large rectangular box moves vertically downward Q.15 The equation of the trajectory is given by
with an acceleration a. A toy gun fixed at A and aimed
towards C fires a particle P. = 240x − x2
(A) 180y = x2 − 240x
(B) 180y

(A) P will hit C if a=g = 135x − x2


(C) 180y = x2 − 135x
(D) 180y
3 . 3 6 | Motion in a Plane

Two projectile are thrown simultaneously in the same Q.21 Statement-I: Three projectiles are moving in
plane from the same point. If their velocities are v1 and different paths in the air. Vertical component of relative
v 2 at angles θ1 and θ2 respectively from the horizontal, velocity between any of the pair does not change with
then answer the following questions time as long as they are in air. Neglect the effect of air
friction.
Statement-II: Relative acceleration between any of the
Match the Columns
pair of projectile is zero.
Q.16 Match the quantities is column I with possible
options from column II. Circular Motion

Particle’s Motion Trajectory Q.22 An object follows a curved path. The following
(A) Constant velocity (p) straight line quantities may remain constant during the motion-

(B) Constant speed (q) Circular (A) Speed (B) Velocity

(C) Variable acceleration (r) Parabolic (C) Acceleration (D) Magnitude of acceleration

(D) Constant acceleration (s) Elliptical Q.23 The position vector of a particle in a circular motion
about the origin sweeps out equal area in equal times-
Assertion Reasoning Type (A) Velocity remains constant

(A) Statement-I is true, statement-II is true and (B) Speed remains constant
statement‑II is correct explanation for statement-I. (C) Acceleration remains constant
(B) Statement-I is true, statement-II is true and statement‑II (D) Tangential acceleration remains constant
is NOT the correct explanation for statement-I
(C) Statement-I is true, statement-II is false.
(D) Statement-I is false, statement-II is true.
Previous Years’ Questions
Q.1 A large heavy box is sliding without friction down
Q.17 Statement-I: The speed of a projectile is minimum
a smooth plane of inclination
at the highest point.
θ . From a point P on the
Statement-II: The acceleration of projectile is constant bottom of the box, a particle 
during the entire motion. is projected inside the box. P
O

The initial speed of the particle



Q.18 Statement-I: Two stones are simultaneously with respect to the box is
projected from level ground from same point with same u and the direction makes
speeds but different angles with horizontal. Both stones an angle α with the bottom as shown in
move in same vertical plane. Then the two stones may the Figure.  (1998)
collide in mid-air. (i) Find the distance along the bottom of the box
Statement-II: For two stones projected simultaneously between the point of projection P and the point Q where
from same point with same speed at different angles the particle lands (Assume that the particle does not hit
with horizontal, their trajectories may intersect at some any other surface of the box. Neglect air resistance.)
point. (ii) If the horizontal displacement of the particle as seen
by an observer on the ground is zero, find the speed of
Q.19 Statement-I: If separation between two particles the box with respect to the ground at the instant when
does not change then their relative velocity will be zero. the particle was projected.
Statement-II: Relative velocity is the rate of change of
position of one particle with respect to another. Q.2 Airplanes A and B are flying with constant velocity
in the same vertical plane at angles 30° and 60° with
Q.20 Statement-I: The magnitude of relative velocity of respect to the horizontal respectively as shown in the
A with respect to B will be always less than VA . figure. The speed of A is 100 3 ms-1. At time t = 0 s,
Statement-II: The relative velocity of A with respect to B an observer in A finds B at a distance of 500 m. This
is given by VAB observer sees B moving with a constant velocity
= VA − VB .
perpendicular to the line of motion of A. If at t = t0, A just
P hysi cs | 3.37


escapes being hit by B, t0 in seconds is  (2014) Q.3 The distance r of the block at time t is  (2016)

A
B
(A)
4
(
R 2 ωt
)
e + e−2ωt (B)
R
2
cos2ωt


(C)
R
2
cos ωt (D)
4
(
R ωt
e + e−ωt )
o o
30 60

PlancEssential Questions
JEE Main/Boards JEE Advanced/Boards

Exercise 1 Exercise 1
Q.15 Q.20 Q.22 Q.3 Q.7 Q.10

Q.25 Q.26 Q.28 Q.15 Q.17 Q.20

Q.29 Q.35 Q.40 Q.22 Q.23 Q.32

Q.41
Exercise 2
Exercise 2 Q.9 Q.12 Q.15

Q.1 Q.5 Q.8 Q.18 Q.20 Q.21

Q.9 Q.11 Q.15

Q.17 Q.27

Answer Key

JEE Main/Boards Q.12 2.0 s


Q.15 25 3 m.
Exercise 1
Q.16 R=4 H1H2
Q.9 10.54 m Q.17 y=2x-5x2

Q.10 10s; 99.1 m s−1 Q.18 653.06 m, 7° 42’ with horizontal, 2.19 s
Q.19 5° 43’
Q.11 58 m s−1 ; 30° 27’ with horizontal; 44.1 m below
and 150 m horizontally away from the starting point Q.20 30° 58’
3 . 3 8 | Motion in a Plane

Q.21 4u2 5g Q.33 π /30 rad. s-2, 5 π /3 cm s-2


Q.22 37 seconds Q.34 987.7 cm s-2
Q.23 1000 km h-1 Q.35 (i) 0.44 rad/s; 5.3 cm s-1 (ii) Not constant, 2.3 cm s-2
Q.24 -3.06ms-2 ; 11.43 s Q.36 0.03 m/s2
Q.25 450 m Q.37 0.86 ms-2
Q.26 40 km h-1 ; 9 min. Q.38 –( xˆ + yˆ )/ 2cm /s2
Q.28 (i) (a) 5km/h, (b) 5km/h; (ii) (a) 0, (b) 6 km/h; Q.39 400 m; girl B
(iii) (a) 1.875 km/h, (b) 5.625 km/h
Q.40 (i) Zero; (ii) 0; (iii) 6×10-3 m/s
Q.29 (i) v0=667 ft/s (ii) 2667 ft (iii) vx=534 ft/s, vy=560 ft/s
5  10 
Q.41 ms-2; β =tan−1  
Circular Motion 7  7 

Q.30 π /1800 rad. s-1 Q.42 ∆v = v( −0.24iˆ + 0.64ˆj)

Q.31 31.4 rad. s-1 Q.43 h=R/3

Q.32 8π rad.s-1 ; 1257.1 cm s-1

Exercise 2

Projectile Motion
Single Correct Choice Type

Q.1 D Q.2 D Q.3 C Q.4 C Q.5 C Q.6 C Q.7 C


Q.8 A Q.9 B Q.10 B Q.11 C Q.12 C Q.13 B Q.14 B
Q.15 C Q.16 A Q.17 A Q.18 A

Circular Motion
Single Correct Choice Type

Q.19 C Q.20 C Q.21 C Q.22 D Q.23 C Q.24 C Q.25 D


Q.26 B Q.27 A Q.28 C Q.29 D Q.30 C

Previous Years’ Questions


Q.1 A Q.2 B Q.3 B Q.4 C Q.5 D Q.6 C Q.7 A

JEE Advanced/Boards Q.3 2.55m, 27o43’


Q.4 (i) 45° , (ii) 2m/sec
Exercise 1 Q.6 4.4 m/s
Q.7 50 m/s
Projectile Motion
Q.8 30.55 m

(1 + α2 )a / 2β Q.9 Yes
Q.1 v=
0
Q.10 (i) 58.8 m (ii) 2gh
Q.2 20 3 m/s, 60° 2 + cot2 θ
P hysi cs | 3.39

Q.11 (i) u=340m/s (ii) 46 s.  


Q.24 ar =−3iˆ − 4ˆjm / s2 ,aN =−4iˆ − 3ˆjm / s2
Q.12 9.8 m, 3.3 m, 1.6s.
5
Q.13 20 s, 2000 m Q.25 fmax = rev / sec

Q.15 (i) 19.6 m/s upward, (ii ) 24.5 m/s at 53° with
Q.26 45°, 2 × 10 4 N
horizontal
40 × 10−2 m s ,a =
Q.27 V = 32 × 102 m s2
Q.16 v 0 t 2(1 − sin α )
Q.17 2.47 m Q.28 0.83 metre on x-axis

Q.19 60° Q.29 0.675 rev/s


Q.20 200m, 20 m/min, 12 m/min Q.30 =
x R cos ωt + vt,=
y R sin ωt,cycloid
Q.21 6 m/s Q.31 334 m
Q.22 tan−1 (1 / 2) 2L
Q.32 2
ω R cos θ
Circular Motion 12
Q.33 2R(asin θ + g − gcos θ)
ω
Q.23 v = 2
θ  v2 
2
Q.34 a + 
R 
 
Exercise 2

Projectile Motion

Single Correct Choice Type

Q.1 D Q.2 B Q.3 B Q.4 C Q.5 D

Multiple Correct Choice Type

Q.6 A, B, C, D Q.7 A, B Q.8 A, B, C, D Q.9 A, C, D Q.10 A, B Q.11 A, C Q.12 B, C, D

Comprehension Type

Q.13 A Q.14 D Q.15 A

Match the Columns

Q.16 A → p; B → p, q, r, s; C → p, q, r, s; D → p, r

Assertion Reasoning Type

Q.17 B Q.18 D Q.19 D Q.20 D Q.21 A

Circular Motion

Multiple Correct Choice Type

Q.22 A, D Q.23 B, D

Previous Years’ Questions


u2 sin2α ucos(α + θ)
Q.1 (i) (ii) (down the plane) Q.2 5 Q.3 D
gcos θ cos θ
3 . 4 0 | Motion in a Plane

Solutions

JEE Main/Boards Let horizontal velocity = v


x
x = vt ⇒t=  ... (i)
Exercise 1 v
Let height of projection= h
Projectile Motion
1 2
Height y = h – gt
Sol 1: Consider two perpendicular unit vectors i, j. At 2
any instant, if we are able to express the position of an 1 x2
y=h– g ∴ y = a – bx2
object w.r.t to initial position as a linear combination of 2 v2
i, j, then we say object is having two dimensional motion g
a = h, b =
in the plane of î , ˆj . 2v 2
Displacement is the shortest distance between two ⇒ Parabolic path
points.
Displacement v2
Velocity = Sol 4: Range r = sin2q
time g
where ‘time’ is time taken to travel between the two It has same value for θ, 90 – θ
points.
→ → → → → 1 → 2 2v sin θ
v = u + a .t; s = u .t + at Sol 5: Time of flight, t =
2 g
→ →
v2 – u2 = 2 a s v 2 sin2 θ
more height h = 2g

Sol 2: We know that, a = ax î + ay ˆj v2


range = sin 2θ
→ g

a = dv
Sol 6: vx = v0 cosθ
dt
→ vy = v0 sinθ – gt where v0 is initial velocity
Let v = vx(t) î + vy(t) ˆj v 0 sin θ – gt
Angle of elevation = tan–1
→ v 0 cos θ

dv dv x (t) dv y (t) ˆ
a = = î + j
dt dt dt gx2
Sol 7: y = x tan θ –
dv y (t) 2v 2 cos2 θ
dv x (t)
⇒ ax= and ay = 2v sin θ
dt dt Time of flight t =
g
→ → 
x = ∫ v.dt + x0
v2
Horizontal range r = sin2θ
→ → g
⇒ x = x0 + ∫ v x (t) dt ˆi + ∫ v y (t) dt ˆj v 2 sin2 θ
More height h =
2g
Sol 3: When a body is projected at an angle in a unit
force field, then the body is called projectile Sol 8: In case of uniform circular motion, the particles
E.g.: Cannon ball shot from a cannon, bullet from a gun, will have acceleration towards the centre only and is
droplets of water coming out from a piston, ball leaving called centripetal acceleration.
a bat etc. v2
ar = = ω 2R
R
P hysi cs | 3.41

Sol 13: Let muzzle velocity be V


2h 2(1.96)
Sol 9: Time of flight t = = = 2 0.1 s Velocity along vertical direction for 60°
g 9.8
50 3
Velocity of bus = 60 kmh–1 = ms–1 v1y = v sin 60° = V
3 2
Velocity along vertical direction for 30°
50
Distance = velocity × time = ( 2 0.1 ) = 10.54 m
3 V
v2y = v sin 30° =
∴ Distance travelled is 10.54 m 2
v 2y
Max height =
2h 2.490 2g
Sol 10: Time of flight = = = 10 s
g 9.8 2
v12 
2
3 
 v1   
Vertical velocity = gt = 10 × 9.8 = 98 ms–1 2g 2
Ratio of heights = =   =   =3
v 22  v2   1 
Total velocity  
2g  2 
= (horizontal velocity)2 + (vertical velocity)2
Hence if shoots thrice as high
= (98)2 + (15)2 = 99.1 ms
–1
v
Time of height t = 2  
g
Sol 11: Vertical velocity vy = gt = 9.8×3 = 29.4 ms–1 3 v 3v
t1 = 2 . =
2 g g
1 2 1
Vertical displacement (y) = gt = × 9.8 × 32
2 2 1 v v
t2 = 2. =
= 44.1 m 2 g g
Horizontal displacements (x) = 50 × 3 = 150 m Their horizontal velocity vx = v cos q
v
v1x = vcos60° =
Final velocity = v 2x + v 2y 2
3
= v2x = v cos30° = v
(50)2 + (29.4)2 = 58 ms
–1
2
horizontal distance = vxt
Total displacement = x2 + y 2 = (150)2 + (44.1)2
v 3v 3 v2
= 156.35 m d1 = v1x . t1 = . =
2 g 2 g
44.1
Angle of depression = tan–1 = 16.38°
150 3 v 3 v2
Its final velocity is 58 ms–1 d2 = v2x . t2 = v. =
2 g 2 g
It is at a distance of 156.35 m at an angle of depression Hence their horizontal range is same.
of 16.38°

Sol 14: Maximum horizontal range occurs when it is


Sol 12: Vertical velocity = v sinq = 9.8 (sin30°)= 4.9 ms–1 fixed at angle of 45°.
1 2 Let its initial velocity be v
h = ut + gt
2
v
1 Vertical velocity vy = v sin 45° =
⇒ 29.4 = 4.9 t + × 9.8t2 ; ⇒ 29.4 = 4.9t + 4.9t2 2
2
 vy  2.v 2v
⇒ 6 = t + t ; ⇒ t2 + t – 6 = 0
2
Time of flight t = 2   = =
 g  2g g
⇒ (t – 2)(t + 3) = 0  
Since, t > 0 v 2y v  1
2
v2
Maximum height (h) = =   . =
⇒ t=2 2g  2  2g 4g
It hits the ground in 2 seconds.
3 . 4 2 | Motion in a Plane

v 2
Horizontal velocity vx = v cos 45° = 1  v2  1 2
2 = 16  sin2θ1  = R
 g  16
 
v 2v v2
Horizontal range (r) = vx.t = . = ∴ R2 = 16H1H2 ⇒ R = 4 H1H2
2 g g
2
v
r g Sol 17: Given that, v = î + 2ˆj
= 2 =4
h v Horizontal displacement x = vx . t ; x=1t
4g
x = t ... (i)
∴ Its maximum horizontal range is 4 times height.
1 2
Vertical displacement y = vyt – gt = 2t – 5t2
2
Sol 15: Horizontal range is given by
⇒ y = 2x – 5x2
v 2 sin2θ
D= where θ is angle of projection v 2 sin2θ
g Sol 18: r = ;
g
sin2(60o )
D1 = (30)2. = 90 sin 120o = 45 3 m v2 80 × 80
10 Max range = = = 640 m
g 10
sin2(30)
D2 = (20)2. = 40 sin 60° = 20 3 m 80 × 80
10 100 3 = . sin2q
10
Distance = D1 – D2 = 45 3 – 20 3 = 25 3 m
sin 2θ = 0.27

Sol 16: For a given angle of projection θ 1 –1


⇒ θ= sin 0.27 = 7.42°
2
v sin2θ 2
Horizontal Range r =
g 2v sinθ 2 × 80
t= = sinθ = 16 sin 7.42° = 2.19 s
g 10
v 2 sin2 θ
Maximum Height h =
2g
given r1 = r2 v 2 sin2 θ
Sol 19: h =
2g
v 2 sin2θ1 v 2 sin2θ2
⇒ = v 2 sin2 θ
g g ⇒5=  ... (i)
2g
⇒ sin2 q1= sin2 q2
⇒ 2q1 = 2q2 or 2q1 = 180 – 2q2 v 2 sin2θ 2v 2 sin θ cos θ
r= =
g g
 q1 ≠ q2
⇒ q1 + q2 = 90°  ... (i) 2v 2 sin θ cos θ
⇒ 200 =  ... (ii)
g
v 2 sin2 θ1
⇒ h1 = Divide (i) by (ii)
2g
v 2 sin2 θ
v 2 sin2 θ2 v 2 cos2 θ1
H2 = = (q1 + q2 = 90°) 5 2g
2g 2g =
200 2v 2 sin θ cos θ
v 2 sin2 θ1 v 2 cos2 θ1 g
H1H2 = .
2g 2g 1 1 1
= tanq; θ = tan–1
4 40 4 10
v
= . sin2q1 cos2q1
4g2 u2 sin2θ 2u2 sin θ cos θ
⇒r= =  ... (iii)
4
g g
v
= . sin22q1
16g2 u2 sin2 θ
⇒H=
2g
P hysi cs | 3.43

r tan θ r Sol 23: Velocity w.r.t ground


= Given = 2 ⇒ tanθ = 2
H 4 H = Velocity w.r.t Jet + velocity of jet
2
4 4u = 1500 km h–1 – 500 km h–1
⇒ sin2θ = ⇒r=
3 5g = 1000 km h-1

gx2
Sol 20: y = x tanθ – Sol 24: v = 126 km h–1 = 35 ms–1
2v 2 cos2 θ
g(50)2 v2 352
13 = 50 tanθ – a= = = 3.0625 ms–1
2 × (10 g)2 cos2 θ 2s 2 × 200
v v 2s 2 × 200
⇒ 26 = 100 tanθ – 25 sec2q ⇒t= = 2 = = = 11.43 s
a v v 35
⇒ 25 tan2θ – 100 tan θ + 51 = 0 (sec2θ = 1 + tan2θ) 2s
2s
100 ± (100)2 – 4 × 25 × 51 Note: - here t = can also be written as
⇒ tan θ = v
2 × 25
s v
100 ± 70 3 17 t= ; is average velocity of motion.
tanθ = = , v 2
50 5 5  
2
3
For tmin , tan θ is minimum ∴ θ = tan–1 ~
– 30.1° So in a uniform accelerated motion,
5
Distance
t=
Sol 21: Given, R=2H Avg velocity

u2 sin2θ u2 sin2 θ Try deriving the same assuming it has some final velocity.
⇒ 2
=
g 2g
⇒ 2sin θ cos θ= sin θ sin θ Sol 25: Distance to be travelled by B relative to A
⇒ tan θ =2 = 2(400) + x
2 1 x = initial separation (distance between them)
sin θ
= , cos θ
=
3 3 Relative velocity = VB – VA = 72 – 72 = 0
2 2 2 2 Relative acceleration = 1 ms–2
So, R= u sin2θ= u 2sin θ cos θ= u 2 × 2= 4u
g g g 3 3g Time = 50
1 2
∵ at = S
Sol 22: He moves 5 steps forward and 3 steps backward 2
in 8 seconds.
1
⇒ He moves 2 m in 8 second ∴ 800 + x = × 1 × (50)2
2
Lets call it drunk movement (D.M) x = 450 m
i.e. in 1 D. M = 2 m in 8 seconds Initial separation was 450 m.
∴ Distance travelled in n D.M = 2n m in 8n seconds.
Now 13m – 5m = 8 m Sol 26: Distance between two busses coming from
same direction is VT; V is speed of bus.
i.e. man will have complete D.M such that
Relative velocity of with buses coming from opposite
2n ≥ 8 for least possible n ⇒ n = 4 direction = V + VC; VC is speed of man.
⇒ he travelled 8 m in 32 seconds Relative velocity of man with buses coming from same
he falls in pit in next 5 m, 5 seconds direction as man = V – VC

∴ He falls in 32 + 5 = 37 seconds. VT VC T
18 = ⇒ 1– =  ... (i)
V – VC V 18
3 . 4 4 | Motion in a Plane

Vt VC T
6= ⇒ 1+ =  ... (ii)
V + VC V 6 Sol 28: (i) 0 to 30
equation (i) + equation (ii) 1 1
distance = V × t = 5 × (t = 30 min = Hr) = 2.5 km
2 2
1 1  distance 2.5
⇒2=T  +  speed = = = 5 km/h
 6 18  time 1/2
2 displacement 2.5
T= = 9 min velocity = = = 5 km/hr
1 1 time 1/2
+ (ii) 0 to 50
6 18
0 to 50 = 0 to 30 + 30 to 50
⇒ V = 40 km/h–1
In 30 to 50
VC = 20 km/h = velocity of cyclist 2
Distance = V × t = 7.5 × = 2.5 km
6
2
2h 2 × 90 (20 min = Hr)
Sol 27: t = = = 3 2s 6
g 10 Displacement (s) = ∑ Vt = 2.5 – 2.5 = 0 km
t = time of descent
Total distance (s) = ∑ | V | t = 2.5 + 2.5 = 5 km
2V1
t1 = s
g Velocity = =0
t
t1 = time of flight between consecutive collisions with
D 5 5
the floor Speed = = = 6 km/hr (50 min = Hr)
t 5/6 6
V1 = (0.9)V (i.e. velocity after collison)
(iii) 0 to 40
V
⇒ t1 = (1.8) ; t1 = (1.8) t 0 to 40 = 0 to 30 + 30 to 40
g
t1 + t2 = t + 1.8t = 2.8t = (2.8) 3 2 ~
– 12 s 1
= 1.25 km
In 30 to 40, distance = v × t = 7.5 ×
6
V12 V2 Displacement = 2.5 – 1.25 = 1.25 km
H1 = = (0.9)2. = (0.81)90 = 72.9 m
2g 20
Total distance = 2.5 + 1.25 = 3.75 km
H 1.25
90 m Velocity = = 1.875 km/h
4/6
72.9 m
3.75
Speed = = 5.625 km/h
4/6

V Sol 29: Vy = V cosθ (θ is angle with vertical)


O 3 2 (1.9) 3 2 12
3 1 2
Vy = V; S = Vy t + gt
Note: - Here we have a sharp change at 3 2 because 5 2
ball got rebound from surface there.
3 1
There is a sharp change at (1.9)× 3 2 because there is 2400 = V(5) + × 32.5 × (5)2 (g= 32.5 ft. /m2)
5 2
change in direction of motion and we took both sides
positive (speed) V = 664.6 ft. s–1

x = Vx. t = V
V
B 4
0.9V sinθ t = .V.5 = 2658.4 ft
5
Vx = V sinθ = 534 ft/s
V
O 3 2 (1.9) 3 2 12 Vy = V cosθ + gt = 560 ft/s
P hysi cs | 3.45

Circular Motion 1 π π
∴a= . m/s2 ;
a= m/s2
Sol 30: Minutes hand of a clock completes one 2 30 60
revolution in one hour i.e. 3600 second

1 Re v Sol 34: Time period T =
So, ω = and 1 revolution = 2π Rad ω
3600 s Given T = 2 s
2π π
ω= rad/s ⇒ ω = × 10–2 rad/s 2π
3600 18 ∴ω= = π rad/s
T
and acceleration a = Rω2 m/s2
Sol 31: A wheel making 300 rotation per minute R = 100 cm = 1 m
and one rotation = 2π rad. ∴ a = p2 m/s2
1 minute = 60 sec
300.2π Sol 35: (i) Given that insect completes 7 revolutions in
∴ω= rad/s; ω = 10π rad/s 100 seconds.
60
7.2π
Sol 32: 4 rotations per second ∴ ω = 7 Rev/100s = rad/s
100
14 π
rotations ω= rad/s
⇒ω=4 and 1 rotation = 2π rad 100
s
ω = 0.44 rad/s and v = Rω
⇒ ω = 4.(2π) rad/s; ω = 8π rad/s
R = 12 cm = 0.12 m
and the velocity of a point on its circumference
v = (0.12) (0.44) m/s2
v = Rω
1 v = 5.3 × 10–2 m/s2
R = 50 cm = m.
2 (ii) Acceleration is not constant. Because the direction of
1 the acceleration vector keeps on changing in direction.
v =   (8π) m/s Hence acceleration vector in circular motion can never
2
be a constant vector.
v = 4π m/s

| a | = Rω2 = (0.12) (0.44)2 m/s2
revolutions 2π
Sol 33: ωinitial = 100 = 100 rad/s →
minute 60
| a | = 2.3 × 10–2 m/s2
10π
ωi = rad/s
3
revolutions 2π Sol 36: Earth completes 1 rotation in 1 day
ωf = 400 = 400. rad/s
minute 60
rotation w
i.e., ω = 1.
40π day
ωf = rad/s
3

ω = 1. rad/s
∆ω ω – ωi 24 × 60 × 60
α= = f
∆t t A
π
40π 10π ω= × 10–2 rad/s
– 432
3 3 30π
α= rad/s2 = and now acceleration at point A;
5 × 60 3 × 5 × 60
a = rω2
π
α= rad/s2 r = 6400 km = 6400 × 103 m
30
and linear acceleration a = Ra r = 64 × 105 m
π2
1 ∴ a = 64 × 105 × × 10–4 m/s2
R = 50 cm = m (432)2
2 A = 0.03 m/s2
3 . 4 6 | Motion in a Plane

5   →
Sol 37: v = 27 km/h = 27 × m/s →
v 2  − 1 (iˆ +ˆj)  v2 ˆ ˆ
(i + j)
18 ⇒ Now a A = aA = –
r  2  r 2
15
v= m/s →
2 Put v = 2 cm/s and r = 4 cm to find a A

v2 (15)2
ar = = = 0.7
R 4 × 80 Sol 39: Now this tests your understanding of

1 displacement vector
Q
a t = 0.5 m/s2 = m/s2
2 → → →
→ → → Displacement vector r = rf – ri
a net = ar + a t = 2
(0.7) + (0.5) 2 →
where rf is the co-ordinates of final A B C
→ →
a net = 0.86 m/s2 position and ri is the co-ordinates of
initial position.
P
Sol 38: At point the acceleration y Now for all the three girls, final
will be centripetal acceleration position is point Q and initial destination is point P. Hence
which is radially directed towards displacement is same for all the three girls,
Q
point O. i.e. A → → → →
→ i.e. 2 r = 2( rQ – rP ) = 2 PQ = 2×200=400 m
2
v ê π/4
Physically: a = (– r ) Distance is the total length of the path travelled.
r O P x
Here for girl B; distance travelled is same as her
Remember êr and êt are the
displacement vector, since she travelled in the straight
unit vectors along radial and line connecting the points.
tangential direction respectively.
Refer to the figure.
Sol 40: Now from the argument made Q
So in this case also, êt →
→ →

2
above, displacement r = rf – ri
→ v êr
a A = r (– êr ) Here the cyclist started from the point P
A O
O and then finally reached the point O
Now, since the point is in O →

between the points P and Q, Hence r f = r i
y →
So r = zero
Hence net displacement is zero.
x Total displacement 0
π/4 And average velocity = = =0
Total time 10

total dis tance


– êr and for average speed =
total time
  π Total distance is |OP| + |PQ| + |QO|
Angle between OA and OP will be
4 |OP| = |QO| = R = 1 m
Now let us resolve (– êr ) into î and ˆj  2πR  π
And |PQ| =  4  =  2  m
π π    
(– êr ) = |– êr | cos (– î ) + |– êr | sin (– ˆj )
4 4  π
∴ Distance =  2 + 2  m.
But since êr and êt are unit vectors;  
| êr | = | êt | = 1  π
2 + 
 2
1 1 ˆ 1 Av. Speed = m/s = 6× 10–3 m/s
 (– êr ) = – î – j =– ( î + ˆj ) 10 × 60
2 2 2
P hysi cs | 3.47

Sol 41: Let us say the circular turn is B Let us say at point B, the A
of the shape AB. particle loses its contact. So N
h B
let us write the equations of
Now at the starting point of the track R cosθ
motions. At point B say the θ θ
→ → → R
i.e. C; a = ar + at particle has velocity v.
O
→ O V
→ a → 2
ar = centripetal acceleration ar mg cosθ = N +
mv

at R mg
v 2 êr C
= (– )
R mv 2
N = mg cosθ –  ... (i)
5 A R
v = 36 km/h = 36 × m/s = 10 m/s
18 Now when the particle is about to lose contact, the
R = 140 m normal reaction between the particle and the surface

(10)2 5 becomes zero.
ar = = m/s2 (– êr )
140 7 ∴ N=0
dv mv 2
and given that = 1 m/s ⇒ mg cosθ =  ... (ii)
dt R
→ dv → Now energy at point A, taking O as reference
∴ at = ( êt ); a t = 1 m/s2 ( êt )
dt 1
EA = 0 + mg R and EB = mv2 + mg R cosq
→ → → 2
Now a = a r + a t Using Energy conservation EA = EB

a = (0.7 (– êr )) + 1 êt ) m/s2 1
⇒ mg R = mv2 + mg R cosq
2
|a| = (0.7)2 + 1 = 0.49 + 1 = 1.49 m/s2 = 1.22 m/s2 ⇒ 2mg R (1 – cosθ) = mv2
 1   10  mv 2
and tan β =   ⇒ β =tan–1   2mg (1 – cosθ) =  … (iii)
 0.7   7  R

Sol 42: mv 2
Putting this value of in eqn (ii)
R
O ⇒ mg cosθ = 2mg (1 – cosθ) ⇒ 3 cos θ = 2
50° V
2 2
O V 40° 90° ⇒ cosθ = ⇒ θ = cos–1  
3 3
40° 40°  2
B and now h = R (1 – cosθ) = R  1 – 
V  3
A R
∴ h= .
→ 3
Velocity at point A, V A = v î

Velocity at point B, VB = v sin 50 î +v cos 50 ˆj Exercise 2

VB = v (0.76 î + 0.64 ˆj ) Projectile Motion
→ →
Now change in velocity DV = VB – V A Single Correct Choice Type

= v (0.76 î + 0.64 ˆj ) – v î Sol 1: (D) Lets solve the


problem taking along
DV = v (–0.24 î + 0.64 ˆj )
plane as x-axis and y x
perpendicular to plane
as y-axis (45°θ)
Sol 43: This is a very standard problem for a JEE
aspirant.
θ 45°
3 . 4 8 | Motion in a Plane

y x V = Vx if Vy(t) = 0
g sinθ
Vy2 42
Vy(t) = 0 ⇒ H = = = 0.8 m
g cosθ 45°θ 2g 2 × 10
But given height is 0.45 m
Vx = V cos (θ – 45o); Vy = V sin (θ – 45o)
∴ |V| > |Vx|
g
gy = g cosφ = ; φ = 45° The given data is sufficient to calculate Vy(t)
2
1 2
g 0.45 = 4(t) – .gt
gx = g sinφ = 2
2 From this we can get t
Vy And Vy(t) = 4 – g(t)
time of flight = 2
gy So data is sufficient.
2V cos(θ – 45) 2 2V sin(45 – θ)
= =
g g 2H
Sol 5: (C) t =
2 g
It hits perpendicularly to plane 1
Horizontal displacement (x) = a t2
⇒ Vx = 0 2 x
ax = horizontal acceleration
⇒ 0 = V cos(θ – 45) – gx(t)
2
1  2H 
2 2V sin(θ – 45)
g  
⇒ V cos(θ–45)= . x = ax  g 
2 g 2  
1 tan θ – 1 1 ax
⇒ tan(θ – 45) = ⇒ = 6
2 tan θ + 1 2 x = H. g = 20 × = 12 m
10
⇒ tanθ = 3
⇒ θ = tan–13
Sol 6: (C)
10 ms–1
Let’s solve the problem along the 45°
Sol 2: (D) Both the bodies have same horizontal range,
plane of inclined surface.
since they form complementary angles with horizontal
axis. y
It θ = 45° they have same time of flight else differs. 10 ms–1 45°
gy
45° gx
Sol 3: (C) u = u cosθ î + u sinθ ˆj x
Let final velocity be v g
gx = gsin45° =
v = v cos α î + v sin α ˆj 2
u cos θ = v cos a g
gy = gcos 45° =
{ Horizontal components of velocity are same} 2

ucos θ V = V cos θ î + V sin θ ˆj


⇒v= = u cosθ seca
cos α 10 10 ˆj
V= î +
3 4 2 2
Sol 4: (C) V = Vcos θ î + Vsin θ ˆj = 5   î + 5  5  ˆj
5  
2.
10
V = 3 î + 4 ˆ
j 2Vy 2
Time of flight = =
|V (t)| ≥ Vx gy g
2
⇒V≥3
Time of flight = 2 sec
P hysi cs | 3.49

Sol 7: (C) Let’s solve along plane of incline. Sol 9: (B) B C


V
V y
53° 127°
gy gx θ
x
2V
Time of flight t = A
gy

1 4
Horizontal range along inclined = g t2 Vy = Vsin53° = 5. = 4 km/hr
2 x 5
1 4.V 2 2V 2 .gx Vx = V cos 127° = – 3 km/hr
= gx . =
2 g2y g2y Resultant velocity along x-direction

gx = g sinq; gy = g cosq = VR + Vx = 4 – 3 = 1 km /hr


width 200
2V 2 .gsin θ 2 Time of swim = = = 180 seconds
r= ; r = 2V tan θ sec θ Vy 5
g2 cos2 θ g 4.
18
1 2  1×5 
Sol 8: (A) Height of monkey above ground = H – gt Drift = VResult × t =   × (180) = 50 m
2  18 
1 2
Height of arrow above ground = Vyt – gt Drift 50
2 Time of walk = = = 60 second
V 5
1 3×
 At point of contact both are at same height H – 18
2
1 Total time = 180 + 60 = 240 s = 4 minutes
gt2 = Vyt – gt2 ⇒ H = Vyt
2
H Sol 10: (B)
⇒t=
Vy
y
H 1 km
Now, here L = Vx . t ; ⇒ L = Vx .
Vy
x
L 
⇒ Vx = Vy  
H 1
Time = 15 min = hr
4
L2 Vy
V = Vy 1 + (V = Vx2 – Vy2 ) = H2 + L2 width 1
H2 H Vy = = = 4 km/hr
time 1
Let vertical velocity at the time of impact be Vf 4
Vf = Vy – gt For shortest path, resultant speed along x-axis = 0
Minimum value of Vt = – Vy ⇒ VR – Vx = 0 ⇒ VR = Vx
∴ – Vy = Vy – gt
Vx2 + Vy2
V=
H
2Vy = gt; t=
Vy ⇒ Vx = 3km/Hr ⇒ Vy = 3km/Hr
g.H
⇒ 2Vy =
Vy
Sol 11: (C)
gH B
⇒ Vy = y
2
V 30°
1 gH 2 2 g(H2 + L2 ) θ
⇒V= . H +L =
H 2 2H
A x
3 . 5 0 | Motion in a Plane

V = V sinθ î + V cos θ ˆj V1 3
= + V
ˆ 2 2 2
VResult = V – Vr = (V sin θ – Vr) î + Vcos θ j
They collide in 2 second
V sin θ – Vr
tan30o = V 3 
V cos θ ⇒  1+ V  (2) = 30
 2 2 2 

1 5 3 sin θ – 5
=
3 5 3 cos θ V1 + 3 V = 30
2

⇒ θ = 60° V2 = 3V
1
⇒ He should steer at 30° w.r.t the line of destination
from starting point ⇒ V1 + 3 ( 3 V ) = 30
1

⇒ V1 = 7.5 ms–1
Sol 12: (C) Car moving north ⇒ wind force acting south. V2 = 7.5 ms–1
Also normal winds are acting due east so flag will point
south-east. V1 is velocity of B
V2 is velocity of A
Sol 13: (C)  y
B C A
45° Sol 16: (A)
45°
x Length of string 1: L1
= x1 + 2x2 + x3
V V ˆ x2
VB w.r.t C = – î + j Length of string 2: L2 = x4 x3
2 2 x1
x1 = x2 + x4
V V ˆ
VA w.r.t B = î + j Differentiate on both side B
2 2 x4
dx1 = dx2 + dx4
V V ˆ V V ˆ
VA w.r.t C = VAB + VBC = î + j – î + j dx4 = 0 ∴ length of string is
2 2 2 2 constant
ˆ ⇒ dx1 = dx2 ... (i)
VAC = 2Vj
VCA = – VAC = – ˆ L1 = x1 + 2x2 + x3
2Vj
Differentiating dL1 = dx1 + 2dx2 + dx3
dl1 = 0 as length of string is constant.
Sol 14: (B) Vrain = Vx î + Vy î
dx1 + 2dx2 + dx3 = 0
Vrain = 2ms–1
⇒ 3dx1 + dx3 = 0 (dx1 = dx2)
Vrc = Vrain – Vcyclist
dx3 3dx1
Vrc = Vy ˆj ⇒ dx3 = – 3dx1 ⇒ =–
dt dt
⇒ Vcyclist = Vx î VB = – 3 VA = – 3(5) = – 15 ms –1

Sol 15: (C) Vx1 Sol 17: (A)


Vx1 = Vx2 Length of string L1= x1+2x3+x2
60°
⇒ V1 sin60° = V2 sin30° xB = x3 + x4 x1 x3 x2

3 V ... (i) dxB = dx3 + dx4


V2 = 1
dx4 = 0 (length of string
Relative velocity linearly 30°
constant) x4
= V1 cos 60 + V2 cos30 ⇒ dxB = dx3
Vx2 B
P hysi cs | 3.51

x1 = xA, x2 = xC
ω1
L1 = xA + 2x3 + xC ⇒ =1
ω2
dL1 = dxA + 2dx3 + dxC
1
dL1 = 0 Sol 20: (C) r = 25 cm = m.
4
dxA + 2dx3 + dxC = 0 And given ω = 2 rev/s
dxA + 2dxB + dxC = 0 But 1 rev = 2π rad
1  ω = 2(2π) rad/s ∴ ω = 4π rad/s
dxB = – (dxA + dxC)
2 1
Now acceleration = rω2 = (4 π)2 m / s2
2
d xB d x
2
d xC2  4
1 A a = 4p2 m/s2
⇒ =–  + 
dt 2
2  dt 2
dt2 
 
lets take upwards as positive Sol 21: (C) P ω
Now acceleration of P is rP.ω 2 rP
1 1
⇒ aB = – (a – f) ∴ aB = (f – a) towards centre of disc and
2 2 acceleration of Q is rQ.ω2
rQ
Given rp > rQ
Sol 18: (A) Length of string L = 2x1 + 2x2 + 2x3
∴ aP > aQ
x1 x2

→ →
Sol 22: (D) Velocity V = r × ω
x3 →
→ →
Acceleration = ( r × ω ) × ω
Now In uniform Circular motion, ω is constant and of
d2L  d2 x 2d2 x2 2d2 x3  course r is constant. Hence magnitude of both velocity
 1 
= 2 2 + +
dt2 dt2 dt2  and acceleration are constant. But the directions keep
 dt varying.
0 = aA + aB + aC Hence both velocity and acceleration change.
aA = 2 and aB = – 1
Sol 23: (C) Equal angles in equal time implies ω is
(B is moving away from central line) constant. Now follow the above argument
aC = – (2 – 1 ) = – 1
∴ aC = 1 ms–2 upwards (A) Sol 24: (C) θ = 2t3 + 0.5
Note: - try understanding the sign convention used here. dθ 2
Positive was towards a reference point and negative was ω= = 6t2 = 6t0
dt t to
away. o

Now here to = 2 s
Circular Motion ω = 6(2)2 = 24 rad/s

Single Correct Choice Type


Sol 25: (D)
Sol 19: (C) This is just a Kinematic problem. So nothing
A seconds hand completes 12 w→
to do with the masses of the bodies. v 1 = rwiˆ
one revolution in A
And now given that both complete a circle in time ’t’. 60 seconds w
9 3 y
B
∴ Both of them have same time period. i.e. 2π rad in 60 seconds
x
T1 = T2 = t 6

2π 2π ∴ω= →
60 v 2 = rw(– ˆj) w
and we know T1 = and T2 =
ω1 ω2
π
= rad/sec
30
3 . 5 2 | Motion in a Plane

Speed of the end point = rω Sol 29: (D)


v at
π π For uniformly accelerated motion;
= 6. cm/s = cm/s velocity will be in tangential ar
30 5
direction. And acceleration
=2π mm/s = 6.28 mm/s will have both the radial and
tangential components.
Now consider the end point at point A; velocity of the
end point would be →

v2 →
dv

ar = and a t =
R dt
v A = rω î and now when the end point is at point B;

velocity of the end point is vB = rω(– ˆj ) Sol 30: (C) F ∝ r
→ → ⇒ F = kr (k is a constant)
Now v A – v B = rω î – rω(– ˆj )
But we also know that for a particle in circular orbit;
→ →
v A – vB = rω( î + ˆj ) mv 2 mv 2
F= ∴ = kr
→ →
r r
| v A – v B | = rω( 2 ) = 2 rw
k
v= r ⇒ v ∝r
= 2 (6.28) mm/s = 8.88 mm/s m

Sol 26: (B) Initially the fan makes 600 revolutions per
minute Previous Years’ Questions
 2π 
∴ ω = 600 rev/min = 600  rad / sec  Sol 1: (A) To cross the river in shortest time one has to
 60  swim perpendicular to the river current.
∴ 1 rev = 2π rad
1 min = 60 sec Sol 2: (B) Shortest possible path comes when absolute
2π velocity of boatman comes perpendicular to river
ωi = 600 rad/sec current as shown in figure.
60
ωi = 20π rad/sec Vr

and finally the fan makes 1200 revolutions per minute



∴ ωf = 1200 rad/sec ; ωf = 40π rad/sec River
60 Vb
Vbr Courrent
Increase in angular velocity = Dω = ωf – ωi
= (40π – 20π) rad/s = 20π rad/s

ω ω 1 1
t= = ; =
Sol 27: (A) Let ‘R’ be the radius of the wheel. In one vb 2
v br – v r2 4 25 – vr2
revolution, the wheel completes a distance of 2pR.
And for 2000 revolutions, it is 2000× 2pR. Solving this equation we get vr = 3 km/h
But given the distance is 9.5 km
 2000 × 2pR = 9.5 × 103 m Sol 3: (B)

∴ R = 0.75 m θ2 > θ1 ∴ ω2 > ω1


Diameter d = 2R = 1.5 m Statement-II, is formula of relative
velocity. But it does not explain
statement-I correctly. The correct
dω explanation of statement-I is due
Sol 28: (C) Angular acceleration α =
dt to visual perception of motion.
Since ω is a constant The object appears to be moving
faster, when its angular velocity is
α = zero
greater w.r.t. observer.
P hysi cs | 3.53

u2  a 
Sol 5: (D) maximum vertical height = = 10m V= (1 + α2 )  
2g
 2β 
u2 sin2θ
Horizontal range of a projectile = Sol 2: Let Vx be velocity along Vy
2g
Range is maximum when θ = 45 0 x-axis. Let Vy(t) be velocity
along y-axis at time t.
u2
Maximum horizontal range = at t = 2, θ = 30° θ=30°
g
Vy (2)
Hence maximum horizontal distance = 20 m. ⇒ = tan30° g
Vx θ0 B
Sol 6: (C) a ∝ r Vx
1
⇒ Vy(2) = Vx
Sol 7: (A) x = t; y = 2t − 5t2 3

Equation of trajectory is y = 2x − 5x2 At t = 3, θ = 0° (given moving horizontal)


⇒ Vy (3) = 0
V – u = at
JEE Advanced/Boards
⇒ Vy (3) – Vy (2) = – g(3 – 2)

Exercise 1 ⇒0–
Vx
= – g ; ⇒ Vx = 3g
3
Projectile Motion Initial velocity of projectile

Sol 1: Let Vx Vy be velocities along x, y axis respectively. V= Vx2 + (Vy (0))2


dVy Vy(2) – Vy(0) = – g (2 – 0)
= – a (given); y = ax – bx2
dt Vx
Vy (0) = Vy(2) + 2g = + 2g = g + 2g
dy d dx dx 3
Vy = = (ax – βx2) = α – 2bx Vy(0) = 3g
dt dt dt dt
Vy = aVx – 2bxVx  ... (i) ⇒V= Vx2 + (3g)2 = ( 3g)2 + (3g)2 = g 12 = 10 12
at origin, (x, y) = (0, 0) V = 20 3 ms–1
⇒ Vy(0, 0) = aVx – 2β(0)Vx Vy (0) 3g
tanq0 = = = 3 ⇒ θ = 60°
Vy = aVx Vx 3g
V2 = Vx2 + Vy2 = Vx2 + a2 Vx2
Sol 3: V = V cosθ î + V sinθ ˆj
⇒V= 1 + α2 Vx  ... (ii) θ = 37°, V = 700 cms–1 = 7 ms–1

Coming back to equation (i) ∴ V = 7 cos37° î + 7 sin 37° ˆj = 7 (0.8) î + 7(0.6) ˆj


dVy d V = 5.6 î + 4.2 ˆj
ay = = (aVx – 2bxVx)
dt dt Distance between the balls along the line of projection
d  dx   dVx  d = 500 cm
– a = α (Vx) – 2bVx   –2bx  
dt  dt   dt  Distance between the balls along x-axis (dx)

d = d cosθ = 500 cos 37° = 500(0.8) = 400 cm = 4m


V = 0  it is having acceleration only in x-direction
dt x When the two balls hit, their x-coordinates are same
 dx  dx
∴ – a = – 2bVx   ∴ – a = – 2β(Vx)2 4
 dt  ⇒t= = s
Vx 5.6
a a
⇒ Vx = ⇒V= 1 + α2
2β 2β
3 . 5 4 | Motion in a Plane

Distance through which ball B falls is 4H 16H2


⇒ sinθ = ; sin2θ =
 4 
2 (4H)2 + R 2 16H2 + R 2
1 1
= gt2 = x 10 ×   = 2.55 m
2 2  5.6  2gH
V2 = (from (i))
sin2 θ
Vy of ball at O = 4.2 ms–1 y
2 × 10 × H 5(16H2 + R 2 )
Vy at time of collision B = =
g 16H 2 4H

cm
V – u = at

0
10H2 + R 2

50
 4 
Vy – 4.2 = – 10   37°
x 5(16H2 + R 2 )
 5.6 
O
V=
103 4H
Vy = –
35
103 Sol 6: V0 = Vx î , V(t) = Vx î + Vy ˆj , Vy = gt
Vy –
35
Angle of inclination = tan–1 = tan–1 = –27.72° 3 3
Vx 5.6 |V(t)| = V = V
2 0 2 x
Ball is directed at an angle 27.72° below x-axis. 2
3 3 
Vx 2 + Vy2 = 2
V ; ⇒ Vx + (gt)2 =  Vx 
2 x 2 
Sol 4: (i)  there is no friction and motion is taking
plane in a horizontal plane, 5 2 2gt
⇒ (gt)2 = Vx ; ⇒ Vx =
4 5
Hence acceleration = 0 in all frames of reference (except
some random accelerating frame of reference which we 1
will not be using in this problem) 2 × 10 ×
2 = 4.4 m/s
=
Vball = Vcos φ î + V sinφ ˆj 5

Vball – trolley (VbT) = Vball – Vtrolley = Vcos φ î + (V sinφ – Vtrolley) ˆj V22 sin2θ
Sol 7: OA = (V2 = 40 ms–1)
Hence motion of the ball is a straight line as observed g
by trolley.
40 × 40 sin120
In trolley’s frame of reference, O moves downward let = = 80 3 m
10
initial position of O be Oo. OoA makes 45° with x-axis.
V 3 ˆj
And the ball follows the path O0A. Hence velocity vector V = V cosθ î + V sinθ ˆj = î + V
of the ball makes 45° with the x-axis in this frame 2 2
θ = 45° OA = Vx t

4θ 4(45) V
(ii) φ = = = 60° ⇒ . t = 80 3
3 3 2
V sin φ – Vtrolley = tan θ = 1 Vt = 160 3
V cos φ 1 2 3 1
V sin60 – ( 3 – 1) y = Vyt – gt = Vt – gt2
∴ =1 2 2 2
V cos60
V 3 V = 160 3
= V – ( 3 –1) ⇒ V = 2 ms –1
t
2 2
y = a 3 t (as they meet at same point)
2 2
V sin2θ 2V 3 1
Sol 5: R = = sinθ cosq a 3t= 160 3 t – (10)t2
g g 2 2
V 2 sin2 θ ⇒ 5t2 + a 3 t – 240 = 0
H=  ... (i)
2g
16 3
H tan θ 4H t>0⇒t=
= ⇒ tanθ = 5
R 4 R
P hysi cs | 3.55

160 3 160 3 Sol 10: (i)


V= = ∴ V = 50ms–1 V
t 10 3
5
V g cos30°
Sol 8: Dy = 1.8 – 0.6 = 1.2 φ
g cos30° 60°
1 2
Dy= – ut + gt 60° 30°
2 30°
1
⇒ 1.2 = – ut + (10)t2
2 Let solve in the planes frame of reference
1 φ = 60 – 30 = 30°
u = V sin30 = 18 × = 9 ms–1
2
⇒ 1.2 = – 9t + 5 t2 V = V cos30 î + V sin30 ˆj
Note: Try to understand the sign convention, here 3V V ˆj
downward is taken positive, V= î +
2 2
1 2 V
Here y = –ut + gt ) 2.
2 2Vy V
2
5t – 9t – 1.2 = 0
2
Time of flight = = =
gy gcos30° 3
g
t~
– 1.96 t=
2V 2
D = Vxt = 18 cos 30t = 30.55 m 3g
2
 
Sol 9: y = x tanθ –
gx2 1 3V  2 V  1  2V 
x = Vxt – gxt2 = g – gsin30  
2(V cos θ)2 2 2  3  2  3g 
θ = 45°  
2
V2 V
Let 4 ft above the ground be taken as plane of referxe = –
g 3g
⇒ y1 = – 4 ft
2V 2 2 (29.4)2
x1 = 350 ft x= = . = 58.8 m
3g 3 9.8
gx12
y1 = x1 tanθ –
2V 2 cos2 θ gx2
Sol 11: (i) y = xtanθ –
gx12 2V 2 cos2 θ
⇒ = x1 tanθ – y1
2V 2 cos2 θ Let’s take A as origin
g x1 tan θ – y1
⇒ = gx2
2
2V cos θ 2
x12 y = x tan (37) –
2V 2 cos2 37
y = = – 3300 m
We have x2 = 320 ft
x = 9400 m
gx22 2 4
⇒ y2 = x2tanθ – 3 g × (94) × 10
2V 2 cos2 θ – 3300 = 94 × 102× –
4 32
2V 2 . 2
 x tan θ – y  5
= x2 tanθ – x22  1 1 
 x 2  10 4 × (94)2 g  3 
 1  = 102  94 × + 33  = 103.5
18  4 
(320)2 V2 .
= 320 tan(45) – (320tan(45°)–(–4)) 25
(350)2
⇒ y2 = 24.08 ft (10)2 × (94)2 × 9.8 25
⇒V= × =340.9 ms–1
(103.5) 18
y2 > 24 ft
∴ It will clear the fence 9400
(ii) t = = 46 s
V cos37
3 . 5 6 | Motion in a Plane

Sol 12: Angle of projection D 58.8


Vy Time of flight t = = =4s
8 Vtruck 14.7
θ = tan–1 = tan–1 = 53°
Vx 6
t 4
u = g   = 9.8 × = 19.6 ms–1 vertically upwards
V= 62 + 82 = 10 ms
–1
2
  2

V2 2 × 10 × 10 3 4 (ii) V = V + V = 19.6 ˆj + 14.7 î


ball bT truck
range = sin2θ = × × = 9.6 m
g 10 5 5
|V| = (19.6)2 + (14.7)2 = 24.5 ms–1
Vy2 (8) 2
max height = = = 3.2 m Angle of projection
2g 2 × 10
Vy
2Vy 2×8 θ = tan–1 = tan–1 19.6 = 53°
Time of height = = = 1.6 s Vx 14.7
g 10
Sol 16: V1 = V0 ˆj
2h 2 × 1960
Sol 13: E = = = 20 s V2 = V0 cos α î + V0 sin α ˆj
g 9.8
V2 = V0 (cos α) î + V0(sinα – 1) ˆj
MN = Vx × t = 100 × 20 = 2000 m
a1 = – g ˆj ; a2 = – g ˆj ; a2 = 0

1
Sol 14: (i) Let initial velocity be V Separation x(t) = V21(t) – a21t2
2
V 2 sin2 α
H= = V0(cos α)t î + V0(sinα – 1)t ˆj
2g
Vy V sin α |x(t)| = t V0 (cos α )2 + (sin α – 1)2
t= =
g g
= V0 .t 2 – 2sinα = V0 .t + 2(1 – sin α )
Horizontal distance = H tanq
Note: - body just gazes ⇒ Vy = 0 at the top. Sol 17: θ1 θ2
H tanθ = Vx .t Let the vertical components of their velocities be Vy. Let
V sin α this angle of depression be q1, q2.
H tanθ = V cosα .
g Vy
tan q1 =
2
V sin α 2
V sin α cos α 2 3
. tan θ =
2g g Vy
tan q2 =
⇒ tan θ = 2cota 4
 They both are perpendicular, q1 + q2 = 90°
u2 sin2 α
(ii) H = (v = u)
2g 1
⇒ tanq2=tan(90 – q1)=cot q1=
tanθ1
2Hg ⇒ tan q1 . tan q2 = 1
u2 =
2
sin α Vy2
u = 2Hg cosec α = 2Hg(1 + cot α)
2 2 2 ⇒ =1
12
  tan θ   gh (4 + tan θ)
2 2
⇒ Vy = 12 ms
–1
= 2Hg  1 +   =
  2   2 Vy
 Vy = gt; t=
g
Sol 15: (i) The ball returns to him
Separation = relative velocity × time = (Vx1 – Vx2) t
⇒ there is no velocity in x-direction in the truck’s frame
of reference Vy 7 × 12
= [3 – (–4)] × = = 2.43 m
⇒ Angle of projection = 90° g 10
P hysi cs | 3.57

Note: Here g is taken 10 ms–2. You may take g = 9.8 Vy


ms–2 them separation = 2.47 m. The questions takes the ⇒ Vy = 3 Vx ⇒ 3 =
Vx
value of g to intelligently manipulate the question.
Vy
angle of projection = tan–1 = tan–1 3 = 60°
Sol 18: Let initial velocity V = Vx î + Vy ˆj Vx

t = seconds V(t) = Vx î + (Vy – gt) ˆj Sol 20: If he takes minimum time ⇒ he is always
Vy Vy – gt perpendicular w.r.t water
tanα = tanβ =
Vx Vx ⇒ drift = velocity of water × time

Vy = Vx tana 120
Vw = = 0.2 ms–1 = 12 m/min
10 × 60
Vx tan α – gt
tanβ = If he takes shortest path, his resultant velocity along the
Vx flow of river is 0 ms–1
gt ⇒ i.e. Vw – Vx = 0 ; Vx = 0.2 ms–1
Vx =
tan α – tan β Lets assume his velocity is V
V= Vx2 + Vy2 = 1 + tan α . Vx
2
V(10) = Vy(12.5)

gt 4
⇒ Vy = V
= 1 + tan2 α . tan α – tan β 5
3
gt ⇒ Vx = V ( V 2 + V 2 = V)
= sec α . 5 x y
sin α cos β – cos α sin β
3
cos α cos β ⇒ 0.2 = V
5
1 gt ⇒ V = 0.33 ms–1 = 20 m/min
= . . cosα cosb
cos α sin( α – β)
width = V × 10 = 200 m
gt cos β
V=
sin(α – β) Sol 21: Velocity of wind = u î
⇒ V butterfly w.r.t earth = V + Vwind
Sol 19: Let V = Vx î + Vy ˆj = (10 + u) î + 12 ˆj
Velocity at maximum height Vh = Vx ( Vy = 0) 12
tanθ =
Vy2 10 + u
Maximum height =
2g 3 12
=
4 10 +u
Velocity at half maximum height = Vy
2
⇒ u = 6ms–1
Vy2 1 Vy2 Vy
2
= ⇒ Vy2 = Note: - The resultant velocity is directed along AB.
2g 2 29 2
2
Sol 22: Vrain/grd = – 20 ˆj 10iˆ
 Vy 
V= Vx2 +   Vm = 5 î
 2
 
Vair = 15 î 20ˆj
Now as per given information,
Vrain – Vground = – 20 ˆj θ
2
2  Vy  ⇒ Vrain = 15 î – 20 Ĵ
Vx2 +   = Vx
5  2 Vman = 5 î
 

5 2 Vy2 ⇒ Vrain/man = 15 î – 20 Ĵ – 5z = 10 î – 20 Ĵ
⇒ Vx = Vx2 +
2 2
3 . 5 8 | Motion in a Plane


10 1
So tanθ = = a r = – 3 î – 4 ˆj m/s2
20 2
→ → →
1 And also a = a r + a t
⇒ θ = tan   –1
2 →
Given a = –7 î – ˆj
Circular Motion → → →
∴ a t = a – a r = (–7 î – ˆj ) – (–3 î – 4 ˆj )
Sol 23: Let the time = t0 at the Bullet →
instant the bullet hits the first a t = – 4 î + 3 ˆj m/s2
disc and makes a hole in it. 
And time = t1 when bullet
makes hole in Disc-2. In this
D1 D2 Sol 25: Acceleration inside a rotor = Rω2
time interval Dt = t1 – t0, →
a = Rω2
An angular displacement of θ is made by point A w.r.t
the point B. →
How for a max
At t = t1 A
2
θ
B
amax = R ωmax
A is hole in Disc-I
Given amax = 10g = 100 m/s2
B is hole in Disc-II
∴ ω Dt = θ → (i) 100 10
ωmax = = rad/s = 5 rad/s
4 2
And also in the same time interval Dt;
1
Bullet travelled a distance of ‘’ we know that 1 rad = rev

∴  = v Dt → (ii) 5
∴ ωm = rev/s
Comparing eq → (i) and (ii); we get
n 2π
ω∆t θ
= Sol 26:
v∆t 
w mv 2
v= N sinθ = ... (i)
θ R
N cos θ = mg → ... (ii)
→ →
Sol 24: r = 3 î + 4 ˆj ; v = – 4 î + 3 ˆj
→ → N
| r | = 5m; | v | = 5 m/s θ
θ mv 2
v2 (5)2
We know that radial acceleration = = = 5 m/s2 R
r 5
And this acceleration will be along the negative radial
direction. θ

∴ r = 3 î + 4 ˆj mg

Unit vector in the direction of r
→ Dividing (i) and (ii)
→ r 1
Is r = = (3 î + 4 ˆj ) v2
|r | 5 ⇒ tanθ =
Rg
3 4 ˆj 5
∴ r̂ = î + ... (i) ⇒ V = 108 Km/h = 108 × m/s
5 5 18
V = 30 m/s
→ v2  3 4 
Now a r = (– r̂ ) = 5  – ˆi – ˆj  R = 90 m
5  5 5 
P hysi cs | 3.59

30.30 21 21
∴ tanθ = =1 umbrella = rev/s = .2π rad/s
90.10 44 44
π 21π
⇒ θ= ω= rad/s = 3 rad/s
4 22
Squaring (i) and (ii) and adding them;
Now for a drop on the Rim; velocity
2
 mv 2  →
N2 (sin2θ + cos2θ) =   + (mg)2
 | v | = Rω
 R 
→ 1
2 | v | =   (3) m/s
 mv 2  2
N= (mg)2 +  
 R 
  → 3
|v|= m/s
2
N = m (10)2 + (10)2
Now this is fairly a kinematics problem;
N = 10 m 2 Newton 3
v= m/s
⇒ N = 104. 2 N. 2

1.5
Sol 27: In solving this question, we will use one of the
most important theorems in circles B
Vx
A θ
A x
θ
r
R Vy
O
C 1 2
2θ 1.5 = 0. t – gt
O 2
B C
 ⇒t= 0.3 and x = vt
The figure explains us that for every θ traversed by AB ,
 3
OB traverses an angle of 2θ. x= × 0.3
2
∴ ωOB = 2 × ωAB x = 0.82 m
Hence in this case ω w.r.t C is twice that of w.r.t point C.  vy 
and tan θ =  

∴ ω = 2(0.4) = 0.8 rad/sec.  vx 
→ vy at point B is vy = 0 – gt
| v | = Rω
1 3
R = 50 cm = m vy = – 10 0.3 m/s and vx = m/s
2 2
→ 1  vy   10 0.3 
|v|= (0.8) m/s ∴ tanθ =   =  
2  vx   3/2 

   
→ → →
| v | = 0.4 m/s and a = a r + a t
 θ= 74.6°

But here a t = 0
Sol 29: Acceleration inside a rotor = Rω2
→ → v  2
∴ a = a r =   (– êr ) →
a = Rω2
R 

Now for a max
Sol 28: Angular velocity of the ω
2
amax = R ωmax

Given amax = 10g = 100 m/s2


3 . 6 0 | Motion in a Plane

cos θ = 0.3
100 10
ωmax = = rad/s = 5 rad/s
4 2 1000
R= = 334 m
1 (10)(0.3)
We know that 1 rad = rev

5 Sol 32:
∴ ωm = Rev/s
2π Fcosθ

θ
Sol 30: From the top view; The θ F=mRω2
insect looks at the particle as Insect
∴ x-co-ordinates of the Particle F sinθ
= R cosθ But θ = ωt Now mRω2 cosθ = ma
∴ x = R cos ωt; y = R sin ωt
R ∴ a = Rω2cosq

∴ r particle,insec t θ 1 2
Now s = ut + at
2
= R cos ωt î + R sin ωt ˆj
1
L=0+ Rω2 cosθ t2
Now 2
→ → →
r particle, observer = r particle,insec t + r insec t, observer 2L
t= 2
Rω cos θ

r particle, observer = vt î
Sol 33: In this case, there will mv 2

∴ rPO = (R cos ωt + vt) î + R sin ωt ˆj be a pseudo force acting on A N R
the body. Now we use Work- Rsinθ
Hence the motion will be a cycloid. Energy theorem, i.e. work ma
B
R cosθ θ
done by all the forces is equal R
to change in kinetic energy. mg
Sol 31: Now we shall follow a v We know that, work done by
standard procedure rather normal force and centripetal
than a clumsy formula to find force is zero. a
the radius of curvature. vx
θ
v Work done by pseudo force =
Let us first find vx and vy at t θ
ma.(R sin θ)
=3s vy WPF = maR sinθ
vx = v0 = 10 m/s î Work done by gravitational force = mg(R – R cos θ)

vy = 0 – gt = – 30 m/s ˆj Wmg = mgR(1– cosθ)


Net work done = maR sinθ + mgR (1–cos θ)
v 
tan θ =  y  ∴ tan θ = 3 ... (i) 1
 vx  ≡ mv2 = Rm(a sinθ + g(1 – cosθ))
  2
Now we need to resolve the gravitational force normal
to the curve at point P. v= 2R(asin θ + g(1 – cos θ))

Hence it is equal to g cos θ and


→ → →
v = v 2x + v 2y = 200 + 900 = 10 10 m/s Sol 34: a net = a radial + a tangential

Let ‘R’ be the radius of curvature,



v2
ar = . (– êr )
R
mv 2
Then; mg cos θ = →
R a t = a ( êt )
2
v
R = gcos θ ; tan θ = 3
P hysi cs | 3.61

2  2 2
→  v2  1  3  1 
| a net |=
2
a +  m/s 2 = .1 .   , [ 20 ]2
R  2  2   2  
  P.E.    = [7.5, 2.5] J

1
K.E = mV2 – PE
2
Exercise 2
1
= × 1 × ( 20 )2 – [7.5, 2.5] = [2.5, 7.5] J
2
Projectile Motion

Single Correct Choice Type Sol 2: (B) Its trivial.

Sol 1: (D) V = 20 cosθ î + 20 sinθ ˆj Sol 3: (B) V1 = V1cosq1 î + V1 sinq1 ˆj

x 3 V2 = V2 cosθ2 î + V2sinθ2 ˆj
t= =
Vx 20 cos θ V1(t) = V1 – gt ˆj
2Vy 2 20 sin θ V2(t) = V2 – gt ˆj
t = time of flight = =
g g
V12 = V1 – V2
3 2 20 sin θ V12 is independent of t.
⇒ =
20 cos θ g dV12
i.e. a12 = =0
dt
3g
sin2θ = ⇒ Trajectory of particle 1 w.r.t particle 2 is straight line
20
along the direction of V12.
3
sin2θ =
2
Sol 4: (C) V12 = V1 – V2
2θ = 120°, 60°
⇒ θ = 60°, 30° = (V1 cos θ1 – V2 cos θ2) î + (V1 sinθ1 – V2 sinθ2) ˆj

 3 1  V1 cos θ1 = V2 cos θ2
⇒ sinθ =  , 
 2 2  ⇒ V12 = (V1 sinθ1 – V2 sinθ2) ˆj

2 20  3 1 This relative velocity is along ˆj .


t=  , 
10  2 2  ⇒ Trajectory of 1 w.r.t 2 is a vertical straight line.
Note: They need not be one above the other because
 3 1 their initial x coordinates need not be same
t =  5, 5
  x = x0 + Vxt
1 3  Vx1 = Vx2
cos θ =  , 
 2 2  ∴ V1cosθ1 = V2 cosq2

1 3 ˆ 3 ˆ 1 ˆ but x θ1 ≠ x θ2 ⇒ x1 ≠ x2
ˆ
V= 20  2 i + 2 j, 2 i + 2 j
 
Sol 5: (D) Vy = V sinθ
Vx = [ 5 , 15 ] ms–1
2Vy
Vy2  
2 2 Time of flight =
H= =
1  3  ,  1   [ t
 2   2   20 ] = [0.75, 0.25]
2
2g 2g Vy2
   M a x i m u m height =
2g
Vy2 1
P.E = mgH= m × g × = m Vy2  V1 sin θ1 = V2 sin θ2
2g 2
3 . 6 2 | Motion in a Plane

⇒ Vy1 = Vy2  gT 
2

2 2  
V12 = (V1 cos θ1 – V2cos θ2) î V sin θ  2  (gT)2 gT2
h= = = =
⇒ Trajectory of one with respect to other is horizontal. 2g 2g 8g 8
gT2 = 8 h

Multiple Correct Choice Type substitute in (i)


t  t
V2 y = 4h   1 – 
Sol 6: (A, B, C, D) (a) Rmax = T  T 
g
2 x = V cosθ t
V
hmax = =H
2g R = V cosθ T
⇒ Rmax = 2H (A) t x x  x
= ⇒ y = 4h  R  1 – 
T R    R 
V2 2V 2
(b) R = sin2θ = sinθ cosq
g g
Sol 8: (A, B, C, D) The equation is same as that of a
V2 projectile equation
H= sin2q
2g ax = 0°
V2  dy 
R y = ax – bx2  Vy = 
= sin2θ dx 
h 2g 
Vy = aVx – 2bxVx
2V 2 sin θ cos θ V2
= sin2q ay = a(0) – 2b(Vx)2 – 2bVx(0)
gh 2g
dVx
4 ax = =0
tanθ = dx
h
4 ay = –2b Vx2
θ = tan–1   (B)
h
–ay
2V sin θ Vx = ay = – g
(c) T = g 2b
g
4V 2 sin2 θ ⇒ Vx =
gT2 = 2b
g
Vy = aVx – 2bxVx
V2
R= sin2θ Vy(0, 0) = aVx – 2b(0)Vx
g
Vy = aVx
4V 2 sin2 θ
20 + mθ = g
g Vy = a
2b
∴ gT2 = 2R + mq (C)
Vy
=a
Vx
2V sin θ
Sol 7: (A, B) T = tanθ = a
g
∴ θ = tan–1a
gT
Vy = V sinθ =
2
Sol 9: (A, C, D) Horizontal distance = Vx × t = 4 × 0.4
1 gT 1 = 1.6 m
y = Vyt – gt2 = t – gt2
2 2 2
Vy = gt = 10 × 0.4 = 4 ms–1
gT2  t   1 – t   Vy 
=  
T  
 ... (i) ⇒ angle of impact = tan–1   = tan–11= 45°
2 T   Vx 
 
P hysi cs | 3.63

1 2 1 h1 sin2 θ1 (3 / 5)2
H= gt = × 10 × (0.4)2 = 0.8 m = = = 9 : 16
2 2 h2 (4 / 5)2
sin2 θ2
Vmin = Vx = V cosq
Sol 10: (A, B) In box frame of
reference, a acts upwards Vmin cos θ1
1 (4 / 5)
a u g = = =4:3
i.e. Resultant acceleration is (g – Vmin cos θ2 (3 / 5)
2
a) downwards . If g = a, resultant
acceleration = 0, P will hit C. Angle bisector is 45° as θ1, θ2 are complementary angles.

For a > g it hits roof


Comprehension Type
for a < g , P will hit CD
Sol 13: (A) V = Vcos θ î + V sin θ ˆj
Sol 11: (A, C) Q Goes up and reaches same point. θ = 53°, V = 50 ms–1

2V 2.(5) ⇒ V = 50 cos(53) î + V sin(53) ˆj = 30 î + 40 ˆj


Time of flight till than t = = =1 s
g 10 ⇒ Vx = 30 ms–1
Final velocity = – 5 + 10(1) = 5 ms–1 downwards |v(t)| ≥ |Vx|
Distance to ground ⇒ min velocity = 30 ms–1
1 Vy = 40 ms–1 but Vy changes with time
h = 5(t – 1) + g(t – 1)2 = 5(t – 1) + 5(t – 1)2
2 ∴ incorrect is (A).
where t is time from starting (total time of flight)
2Vy 2 × 40
Distance travelled by P = 2H (given) Sol 14: (D) Time of flight (T) = = =8s
g 10
1 2
2H = 5t + gt Now observe the vertical motion, the body ascends
2
then it descends.
⇒ 2(5(t – 1) + 5(t – 1)2)= 5t + 5t2
Now if we observe descent in reverse time, it looks like
10t – 10 + 10t2 – 20t + 10 = 5t + 5t2 ascent,
5t2 – 15t = 0 Hence tascent = tdescent (reverse time)
⇒ t = 0, t = 3; t > 0 ⇒ t = 3 tascent = T – tdescent ⇒ tdescent = T – tascent
H = 5(3 – 1) + 5(3 – 1) = 30 m
2
T=8
⇒ td = 8 – Ta
V2
Sol 12: (B, C, D) R = sin2q ⇒ ta + td = 8 (0 < (ta, td) < 8)
g
∴ All A, B, C satisfy
(70)2 4800
480 = sin2q ⇒ sin 2θ = = 0.96
10 4900
Sol 15: (A) trajectory equation
2θ = sin–1 0.96 = 74°
θ = 37°, θ2 = 90 – θ = 53° (Complimentary angles) gx2
y = x tanθ –
2(V cos θ)2
θ , θ2 are complimentary as they have same horizontal
range. θ = 53°, V = 50 ms–1
2V sin θ 10 . x2
Time of height t = y = x tan53° –
g 2(50 cos53)2
t1 sin θ1 sin37 3
= = = 4 10x2
t2 sin θ2 sin53 4 y = x  –
3
2
 3
2  50 × 
V 2 sin2 θ  5 
Max height h =
2g ⇒ 180 y = 240 x – x2
3 . 6 4 | Motion in a Plane

Since it is single correct you may as well solve by substituting Sol 20: (D) VAB = VA – VB
any 3 points in motion. You may also eliminate option B, D
VAB > VA
as coefficient of x2 should be negative, which is common
knowledge to be known about trajectory equation. ⇒ VB < 0
Which is possible

Match the Columns Hence statement 1 false.

Sol 16: A → p; B → p, q, r, s; C → p, q, r, s; D → p, r
Sol 21: (A) Statement-II true
(A) Constant velocity ⇒ same direction ⇒ straight line
 Relative vertical acceleration is zero, relative vertical
Answer is (A) (B) velocities don’t charge.
(B) Constant speed ⇒ Constant magnitude of velocity
⇒ Variable direction of velocity ⇒ there is acceleration Circular Motion
⇒ It can follow any path.
Sol 22: (A, D) In a curved path; the direction of velocity
B → p, q, r, s →
(C) With variable acceleration, it can follow any path keeps on changing. So v cannot remain constant under

C → p, q, r, s any conditions. However | v |
(D) Consider a particle moving in circle with uniform = Speed can remain constant.
velocity u.

mu2 → dv
Magnitude of acceleration = , directed toward canter. And a = ; so it follows that acceleration also cannot
r dt

This acceleration has constant magnitude, but variable → dv
remain constant. But still | | =
a is possible
direction. dt
Hence (q) false
Sol 23: (B, D) For a circular motion
Now circle is a special case of ellipse
Sweeping equal area in equal time is
⇒ (s) is also false
only possible when ω is constant.
Straight line is a trivial example of constant acceleration.
→ → →
So p is true. ∴ Now v = r × ω
We know that trajectory of a projectile is parabola. Here So velocity is not constant
acceleration is constant g towards ground.

Hence r is true. But speed = | v | = rω = constant
D → p r. → → →
and a = ar + at
Assertion Reasoning Type → dv dω
at = = r. = zero
Sol 17: (B) Speed of projectile is minimum because dt dt
Vy = 0 → → → →
and a r = r × ( r × ω )
∴ Acceleration is not constant.
Sol 18: (D) Vx = V cosθ
 Angle of projections are different,
Vx ≠ Vx are θ1 ≠ θ2
1 2

so they do not collide

Sol 19: (D) Consider two particles in circular motion


P hysi cs | 3.65

Previous Years’ Questions Therefore, u cos (α + θ) – v cos θ = 0


ucos(α + θ)
Sol 1: (i) Accelerations of particle and block are shown or v = (down the plane)
cos θ
in figure.
Sol 2:
 
i j


 gcos  in
in gs
gs 


Acceleration of particle with respect to block The relative velocity of B with respect to A is perpendicular
to line
= (Acceleration of particle) – (acceleration of block)
of motion of A.
= (g sin θ î + g cos θ ˆj ) – (g sin θ) î = g cos θ ˆj Now
∴ AB V cos30° = VA
motion of particle with respect to block will be a
projectile as shown. ⇒ VB = 200 m/s
And time t0 = (Relative distance) / (Relative velocity)

u 500
Q = = 5sec
 VB sin30o
P R

 g cos 
Sol 3: (D)
The only difference is, g will be replaced by gcosθ.
dv
v = ω2r , where v is the velocity of the block radially
u sin2α u2 sin2α
2 dr outward.
PQ = Range (R) = PQ =
gcos θ gcos θ
v r
∫0 vdv = rdr ω2 ∫
(ii) Horizontal displacement of particle with respect R /2
to ground is zero. This implies that initial velocity
with respect to ground is only vertical, or there is no R2
horizontal component of the absolute velocity of the ⇒ v = ω r2 −
4
particle.
u sin(+) r dr r
u ∫R /2 2
= ω∫ dt
0
R
j a r2 −
 vcos 4
i
u cos(+) 
v = r
R ωt
4
e + e−ωt ( )
  v sin

Let v be the velocity of the block down the plane.


Velocity of particle

= u cos (α + θ) î + u sin (α + θ) ˆj

Velocity of block = – v cos θ î – v sin θ ˆj


∴ Velocity of particle with respect to ground

= {u cos (α + θ) – v cos θ} î + {u sin (α + θ) – v sin θ} ˆj


Now, as we said earlier that horizontal component of
absolute velocity should be zero.

Potrebbero piacerti anche